You are on page 1of 89
JNDEX FOR STUDY QUESTIONS Subject General principles (optional) Cell injury Inflammation Fluids and hemodynamics Nutrition Genetics (optional) Neoplasia Hematology: RBC/WBC Lymphoproliferate Coagulation Blood bank Cardiovascular Respiratory Gastrointestinal Hepatobiliary/pancreas Renal/LUT Gynecology Endocrine Musculoskeletal Skin CNS and special senses Answers and explanations Pages 7 8-9 10-11 11-12 13-21 21-22 23-25 25-29 29-31 30-34 35-38 38-41 41-44 45-48 49-53 53-58 58-62 62-69 1 2 2-3 3. 69-88 Number questions i one = worn 14 16 13 17 21 16 14 17 22 20 20 35 342 Note: This material is copyrighted. All rights reserved. (Edward F. Goljan, M.D) 2001 HIGH YIELD STUDY QUESTIONS IN PATHOLOGY Total questions: 328 Optional: General principles in laboratory medicine questions: this material is covered in the Pathology High Yield pages 5-10 1 ‘Assuming the use of 2 standard deviations to establish the reference interval of a test, in a test with ‘reference interval of 10-30 mg/dL, 1 standard deviation would equal. A 25 B. 50 c 75 D. E Ifthe prostate specific antigen (PSA) test for prostate cancer is lowered from a reference interval of 0-10 ng/mL to 0-4 ng/mL, this will... ‘A. increase the number of false negatives B. decrease the number of false positives C. increase the test's specificity D. increase the PV: E. increase the PV" Study the following schematic involving a control group and disease X. Normal _ Disease X Ax Reference interval Vhich ofthe following correctly describes test results in the space occupied by each of the lettered groups? ‘A. Group A: true negatives + false negatives B, Group B: true negatives + false positives C. Group C: true positives + false positives D. Group D: true positives + false negatives ‘A pregnant woman in her frst trimester complains of heat intolerance and palpitations Physical a rtiveals an enlarged, non-tender thyroid gland. Her serum T, is elevated and the TSH is ‘hormal. Which of the following best explains the thyroid function study results? ‘A. Thyroid binding globulin is increased B. Free T, hormone levels are increased C.__ Estrogen increased the synthesis of thyroid hormone D. _ Progesterone increased the synthesis of thyroid binding globulin E.__Patienthas Grave's disease Note: This material is copyrighted. All rights reserved. (Edward F. Goljan, M.D.) 2001 Cell Injury questions |. Hyperplasia is primarily operative in which ofthe following growth alterations? Appearance of the affected kidney in renovascular hypertension Thickened bladder wall in a patient with urethral obstruction Barrett's esophagus in a patient with gastroesophageal reflux Enlarged left atrium in a patient with severe mitral stenosis, Galactorthea in a woman with a prolactinoma moo Which of the following disorders is an example of coagulation necrosis? A. Lobar pneumonia in an alcoholic B. Hepatic abscess in a patient with amebiasis CC. Pseudomembranous colitis in a patient on ampicillin D. Diminished brain mass in a patient with Alzheimer’s disease Embolus to the superior mesenteric artery leading to bowel infarction m 3. Inwhich of the following diseases would you expect a low arterial PO, and a low oxygen saturation (Ss092 Carbon monoxide poisoning Iron deficiency anemia Decreased cardiac output Respiratory acidosis Cyanide poisoning roOm> 4. Which of the following disorders is an example of metaplasia? ._ Increased goblet cells in the mainstem bronchus of a smoker ‘Squamous epithelium in the mainstem bronchus of a smoker C. Proliferative endometrial glands in a woman on unopposed estrogen D. _ Hyperkeratosis of the skin in a patient with psoriasis E, Multinucleated giant cells in a granuloma o> Inflammation questions 1. A 4-year-old child with recurrent Staphylococcus aureus infections and an absent respiratory burst MOST LIKELY has a/an... defect in spectrin in the cell membrane defect in microtubule polymerization deficiency of IgG gamma globulins deficiency of NADPH oxidase deficiency of myeloperoxidase POOw> Which of the following is MOST responsible for the ant-inflammatory activity of corticosteroids? A. _ Increased leukocyte adhesion to endothelial cells B. _ Inhibition of phospholipase A? Cc. Destruction of eosinophils D. Inhibition of cyclooxygenase E. _ Inhibition of lipoxygenase Note: This material is copyrighted, All rights reserved. (Edward F. Goljan, M.D.) 2001 Ttems 3-5 A. Cellulitis B. — Suppurative inflammation C. _ Fibrinous inflammation D. Pseudomembranous inflammation Ez Granulomatous inflammation 3, _AS2.year-old man with chronic renal failure has chest pain and a scratchy 3 component sound heard over the anterior chest 4. Afebrile 8-year-old child, who has not received any immunizations, has 2 srayish-white exudate in ‘the oropharynx and prominent cervical lymphadenopathy 5. A febrile 3 year old child has a diffuse, raised, hot red area of inflammation extending around the right periorbital tissue leading to swelling and closure of the eye ‘A newborn child has failure of separation of the umbilical cord. Histologic. sections of the Surgically removed cord reveal an absence of neutrophil margination and emigron into the sare ital tissue. The clinical and histologic findings in this case are MOST CLOSELY associated with a defect in. ‘A. activation of the complement system B. microtubule polymerization CC. the respiratory burst mechanism D. _anadhesion molecule in neutrophils E, _ the production of myeloperoxidase ‘Neutrophils Macrophages Eosinophils Mast cells . Plasma cells 7. They represent the epithelioid cells and multinucleated giant cells ina granuloma POOwP 8 C3aand CSa activate these cells 9. The granules of these cells contain reffactile material that form Charcot-Leyden crystals in the sputum of asthmatics Fluids and hemodynamics questions: 1. Which of the following characterizes early endotoxic (septic) shock rather than hypovolemic or cardiogenic shock? A. Warm skin B. Decreased cardiac output C. Low mixed venous oxygen content D. _ Increased total peripheral resistance E, Decreased venous return to the heart Note: This material is copyrighted. All rights reserved. (Edward F. Goljan, M.D.) 2001 2. Which of the following edema conditions represents a transudate secondary to an decrease in ‘oncotic pressure? Patient with pneumonia who has a pleural effusion Patient with cirthosis who has dependent pitting edema Patient with edema of the arm post-modified radical mastectomy Patient with a pulmonary infarction who has a left pleural effusion Patient with congestive heart failure with bilateral pleural effusions POO D> 3. In treating a patient with right heart failure who has dependent pitting edema, which of the following would be the MOST APPROPRIATE management of the patient's sodium and water intake? Sodium intake Water intake A. Nochange Decrease B. Increase Increase C. Decrease No change D. Decrease Decrease E, Nochange No change 4. A 58-year-old man with a small cell carcinoma of the lung presents with mental status abnormalities. A CT scan of the brain reveals cerebral edema but no space occupying lesions. Serum electrolytes exhibit a serum sodium of 110 mEq/L (136-145 mEq/L). There is no evidence of pitting edema or volume depletion. Which of the following is the BEST non-pharmacologic ‘treatment of this patient? Sodium intake Water intake A. Decrease Decrease B. Increase Increase C. Decrease No change D. No change Decrease E. —_Nochange No change 5. A 62-year-old man has a ruptured abdominal aortic aneurysm with retroperitoneal hemorrhage. Which of the following Swan-Ganz catheter findings BEST represents this patient's clinical disorder? MVO,; = PCWP. svR Cardiac output A. Normal Normal = Normal Normal B. Decreased Increased Increased Decreased C. Decreased Decreased Increased Decreased D. Increased Decreased Decreased Decreased MYO, = mixed venous oxygen content, PCWP = pulmonary capillary wedge pressure (measure of lft ventricular endiatolic ‘olume), SVR = systemic vascular resistance (peripheral arteriolar resistance) Note: This material is copyrighted. All rights reserved. (Edward F. Goljan, M.D.) 2001 Ttems 6-12 POsm ICF ECF ‘Volume ach set of options relating to volume and osmolality changes in the extracellular and intaceuley fluid compartments is followed by several numbered items. For each numbered item, select the ONE lettered pti that is most closely associated with it. Each lettered option may be selected ono, ore than once, or not at all. Note: the height of the squares represent plasma ‘osmolality (POsm), while the width of the squares represents ‘yolume in each compartment. ECF is ‘the extracellular fluid compartment and ICF is the intracellular fluid compartment. ‘The dark lines represent the ‘normal POsm and volume in each compartment, while the hash marked lines represent the volume alteration. EB 6, The patient is a 28 yr. old man who is volume depleted secondary to traveler’s diarrhea contracted ‘while on a vacation outside the United States 7, ‘The patient is a 58 yr. old man with a long history of ischemic heart disease. He now presents with pibasilar rales and dependent pitting edema. He is not taking diuretics. &, The patient is a 29 yr old man who recently was hospitalized with head trauine secondary to a rae Poyele accident. He now has polyuria and complains of excessive thirst throughout the day. ‘The physical exam is normal. Note: This material is copyrighted. All rights reserved, (Edward F. Goljan, MD.) 2001 9. 10. IL 12, 13. ‘The patient is a 45 yr old man who received excessive amounts of sodium bicarbonate during a cardiac resuscitation. He now has bibasilar rales, neck vein distention, and dependent pitting edema. A179 yr old woman received excessive amounts of 0.9% normal saline during repair of a femoral head fracture, She now has bibasilar rales, neck vein distention, and dependent pitting edema. ‘A.52 yr old man has been taking a loop diuretic and now presents with signs of volume depletion. ‘A.29 yr old man collapsed after finishing a marathon on a hot, humid day. His runing companion states that he did not take any fluid supplements and had been sweating profusely throughout the race. Physical exam reveals signs of volume depletion, ‘A volume depleted patient with type I diabetes mellitus presents to the emergency room in diabetic ketoacidosis. Which of the following changes in electrolyte and volume status is most likely present in this patient? POsm High High High Low Low BOOM E PNa® Low Low High Low Low ECF Compartment Contracted Expanded Expanded Expanded Contracted ICF Compartment Contracted Contracted Contracted Expanded Expanded ECE = extracellular fluid, ICF = intracellular fuid, PNa” = plasma sodium, POsm = plasma osmolality 14, 15 and in an arterial thrombus? A. FactorV B. Platelets C. White blood cells D. Factor VIN E, Prothrombin Which of the following is a blood component that is utilized in both an occlusive venous thrombus ‘A 29 yr. old man sustains bilateral femoral bone fractures and multiple pelvic fractures from a mountain bike accident. Forty eight hours later, he develops a sudden onset of dyspnea, petechial lesions, and mental status alterations. Laboratory studies reveal hypoxemia and thrombocytopenia. ‘The mechanism for this patient's clinical disorder is most closely associated with... A. disseminated intravascular coagulation B. pulmonary embolism C. fat embolization D. air embolization E. pneumonia Note: This material is copyrighted. All rights reserved. (Edward F. Goljan, M.D.) 2001 Items 16-21 pH PaCO2, HCO; (735-745) 3-45 mmHg) (22-28 mEq/L) A 722 69 a B. 7.26 26 rm Cc 733 68 34 D. 7.42 2 4 E751 48 38 F756 24 21 16. AS6 year old patient who has been vomiting 17. AS5l-year-old woman with rheumatoid arthritis has salicylate intoxication 18, Patient with an anxiety attack 19. Patient with ethylene glycol poisoning 20. Patient with cystic fibrosis 21. Patient with barbiturate poisoning Items 22-27 Serum sodium | Serum potassium | Serum chloride / Serum bicarbonate (136-145) G5-5.0) (95-105) (22-28) [ A 118 3.0 8 21 B. 152 28 110 aaa C. 130) 29 80, 36 D. 128) 58 96 18 E. 140) 3.0 15 15 F. 140 5.0 95 15 22, Patient with primary aldosteronism 23. Patient with Addison's disease 24, Patient with diarrhea 25, Patient with an increased anion gap metabolic acidosis, 26. Patient with inappropriate ADH syndrome 27. Patient with excessive vomiting or patient taking loop diuretics who is volume depleted Note: This material is copyrighted. All rights reserved. (Edward F. Goljan, M.D.) 2001 Nutrition questions 1 ‘A 25-year-old woman has not had her period for the last 8 months. She is 5! 2" and weighs 90 pounds. A urine pregnancy testis negative. She states that she has been trying to lose weight for her upcoming wedding. You order a battery of tests and give the patient an intramuscular injection of progesterone. Ten days later the patient returns to your office and reports that she had no withdrawal bleeding. Laboratory tests reveal the following: serum prolactin is normal, serum FSH and LH are low, serum TSH is normal, serum estradiol is low, and serum cortisol and growth hormone are increased. Based on these findings, you strongly suspect the patient has. primary ovarian disease hypopituitarism secondary hypothyroidism weight loss syndrome Cushing's syndrome moomD> Follicular hyperkeratosis, night blindness, and a hemorrhagic diathesis are expected in a patient... with cystic fibrosis with scurvy with hypothyroidism who is a pure vegan who is bulimic mOOM> Which of the signs or symptoms characterize a fat soluble rather than a water soluble vitamin deficiency? Perifollicular hemorrhage Bone pain and tetany Peripheral neuropathy Ophthalmoplegia, confusion, and ataxia ‘Hyperpigmentation in sun-exposed areas mOOR> Which of the signs or symptoms are more prominently found in marasmus than kwashiorkor? ‘Hepatomegaly Pitting edema Reduced total lymphocyte count Flaky paint dermatitis Broomstick extremities PUOB> Pellagra will MOST LIKELY develop in a patient.. ‘who is taking isoniazid whose diet primarily consists of com who is a pure vegan who is taking nicotinic acid to lower lipids with maldigestion secondary to chronic pancreatitis mouODD> Note: This material is copyrighted. All rights reserved. (Edward F. Goljan, M.D.) 2001 10, ML. Which of the following is more often associated with anorexia nervosa than bulimia nervosa? A. Hypokalemia B. Metabolic alkalosis C. Normal body image D. Normal serum gonadotropins E. Osteoporosis Which of the following laboratory test abnormalities would you MOST expect in a patient with morbid obesity? A. Increased serum TSH B. Increased 24-hr urine for free cortisol C. Increased fasting glucose D. Increased 24-hr urine for 17-ketosteroids E. _ Increased serum DHEA-sulfate Which of the following vitamin deficiencies would you expect in a child maintained on unfortified goat's milk? A. Ascorbic acid B. Thiamine C. Niacin D. Riboflavin E, Folate Which of the following vitamins would be deficient in a newborn child with anemia whose mother is a pure vegan? A. Ascorbic acid B. Thiamine C. Niacin D. Pyridoxine E Ba A 65-year-old woman complains of bleeding gums after brushing her teeth, easy bruising, and pain in her legs when walking. Her platelet count is normal. The pathogenesis of her disease is MOST CLOSELY related to... a deficiency of ATP a cofactor deficiency in collagenase lack of hydroxylation of lysine and proline a cofactor deficiency in lysyl oxidase platelet dysfunction HOOD p ‘A.30 year-old man develops an acute onset of confusion, ataxia, nystagmus, and ophthalmoplegia shortly after the administration of an intravenous solution containing 5% glucose and normal saline. ‘The pathogenesis of this patient's neurologic disorder is most closely related to. ‘A. central pontine myelinolysis B. thiamine deficiency C. Purkinje cell atrophy D. _ viral encephalitis E. Bizdeficiency Note: This material is copyrighted. All rights reserved. (Edward F. Goljan, M.D.) 2001 Optional Genetics questions: most of the genetics material is located in Pathology High Yield (pages 11-16). The material will be covered by a geneticist. 1. A551 year old woman delivers a full-term baby that has repeated vomiting of bile stained material. A flat plate of the abdomen reveals air in the stomach and proximal duodenum and no air in the remainder of the bowel. The maternal serum a-fetoprotein level is low. The baby has 46 chromosomes. The mechanism of the child's disease is most closely associated with.. A. aMendelian disorder B. a Robertsonian translocation CC. nondisjunction in meiosis D. a point mutation of a nucleotide E, _amicrodeletion disorder 2. If an African American woman with sickle cell disease has children with a man lacking the abnormal B-chain, you would expect... 25% of their children to have sickle cell trait 50% of their children to have sickle cell trait 25% of their children to have sickle cell disease ‘50% of their children to have sickle cell disease all of their children to have sickle cell trait mOOmD 3. A 17-year-old adolescent presents with primary amenorrhea. Physical exam reveals normal secondary female sex characteristics. Discrete masses are noted in both inguinal canals. A speculum exam of the vagina indicates a blind pouch. You would expect this patient to also have... A. aprostate gland B. seminal vesicles C. anandrogen receptor deficiency D. _ one Barr body on a buccal smear E. _ ovaries in the inguinal canal 4. While examining a 13 year old boy during a routine physical examination, you note bilaterally enlarged, non-tender testicles that do not transilluminate, a high arched palate, and a mid-systolic ejection click followed by a short murmur. You call the school counselor and find that the child has a moderately severe attention deficit syndrome. Which of the following studies would you recommend on this boy that would best explain all of the abnormalities noted on the examination? A. Echocardiogram B. Buccal smear C. Serum gonadotropins D. Identification of triplet repeat E. Chromosome study on his father 5. Prader-Willi and Angelman's syndrome have different clinical features, however they both share a defect at the same location on the same chromosome. This is an example of... ‘A. variable expressivity B. a Robertsonian translocation. C. genetic heterogeneity D. genomic imprinting E. _abalanced translocation 10 Note: This material is copyrighted. All rights reserved. (Edward F, Goljan, M.D.) 2001 6. If the carrier rate for the sickle cell abnormality is 1 in 12, the prevalence of sickle cell disease is approximately | in... A 144 B. 288 Cc. 376 D. 720 E. 1440 7. Which one of the following transplacental infections is associated with sensorineural hearing loss, blindness, and periventricular calcifications? A. Cytomegalovirus B. Toxoplasmosis C. Syphilis D. Herpes genitalis E, Rubella 8. A pregnant woman during her fist trimester developed fever, a maculopapular rash, arthritis, and painful postauricular lymphadenopathy. Which of the following complications could potentially ‘occur in her newborn child? A. Periventricular calcification B. Saddle nose deformity CC. Sensorineural hearing loss D. Limb hypoplasia E. Craniofacial abnormalities Neoplasia questions 1. In which of the following sites is the MOST COMMON primary cancer an adenocarcinoma? SELECT 2 A. Esophagus B. Bladder C. Larynx D. Cervix E. Lung 2. Which of the following genes regulates kinases in the cell division cycle, hence assuming an important role in the development of human cancer? A. ceras proto-oncogene B. etbB proto-oncogene CC. Rb-I suppressor gene D. c-myc proto-oncogene E. _p-53 suppressor gene A primary cancer is more common than metastasis in which of the following sites? A. Lymph node B. Colon Cc. Brain D. Bone E. Lung uw pa ruc, ins materia 1s copyrighted. All nights reserved. (Edward F. Goljan, M.D.) 2001 4. ASS year old woman with breast cancer has an infiltrating ductal carcinoma that is 2 em in size, ERA and PRA positive, metastatic to 5 out of 20 axillary lymph nodes, and metastatic to both the vertebral column and liver. Which of the following MOST influences the ultimate prognosis in this patient? A. Herage B. ERA/PRA status C. Size of the tumor D. Axillary node involvement E. Bone and liver involvement 5. Which of the following is the second most common cancer and cancer killer in men and women? ‘A. Malignant melanoma B. Lung cancer C. Colorectal cancer D. Malignant lymphoma E. Stomach cancer 6. A tumor that could potentially produce Cushing's syndrome and hyponatremia is most likely located in the. A. kidney B. placenta C. liver D. thyroid BE. hung 7. Both AFP and B-hCG are most likely to be elevated in which primary tumor site? A. Lung B. Testicle C. Liver D. Colon E. Ovary 8. Which of the following cancers is prevented by immunization with a commonly used vaccine? A. Pancreatic carcinoma B. Stomach carcinoma C. Transitional cell carcinoma D. Hepatocellular carcinoma E. Cervical carcinoma 12 Note: This material is copyrighted. All rights reserved. (Edward F. Goljan, M.D.) 2001 Hematology questions L ‘A 19-year-old African American woman presents with fatigue and exercise intolerance. She has @ stow of menorthagia, A CBC reveals a mild mieroeytic anemia, 2 low normal WBC count, and a normal platelet count. A corrected reticulocyte count is <2%. Which of the following sets of Tnboratory data best represents the hematologic findings in this patient? ‘Serum fron | TIBC | % Saturation | RBC Count_| Ferritin A. [normal normal | _normal Thigh, ‘normal B. [low Tow Tow low high c. Dow high low, Tow Tow D. [high low Thigh normal high E. [normal ‘normal | normal low normal Which of the following characterizes pernicious anemia rather than Biz deficiency from other causes? ‘A. _ Increased plasma homocysteine levels B. Increased urine methylmalonic acid C. Decreased vibratory sensation D. _ Hypersegmented neutrophils E. High serum gastrin levels |A 28-year-old man has a motoreycle accident outside the emergency room. Physical exam the Sinergence room reveals a weak, tready pulse, cold clammy skin, and a blood presse of 60/40 Sam He. An open right femoral fracture is present as well as tendemess over the left lower ribs. Which of the following would you expect in this patient if a blood sample was drawn prior 0 insertion of an IV line with 0.9% normal saline? SELECT 3 ‘A. Normal hemoglobin and hematocrit B. Normal effective arterial blood volume C.__ Increased central venous hydrostatic pressure D. Decreased mixed venous oxygen content E. Decreased pulmonary capillary wedge pressure ‘Ad year old African American child with sickle cell anemia presents with a high fever and nuchal figidity, Scleral icterus and hepatosplenomegaly are present. A CBC reveals a ‘moderately severe ormocytic anemia and a WBC count of 21,000 cells/uL with left shift. A urinalysis is normal. A. vpinal tap reveals turbidity, increased protein, and neutrophils. You would expect a gram stain of CSF and a blood culture to reveal... ‘A. gram positive diplococci B. grammnegative diplococci C. gram positive cocci D. gram positive rods E. gram negative coccobacilli B EEE ese TE ol hn TT ss ie i i hi ‘Noie: { his material is copyrighted, All rights reserves. (Edward F. Goljan, M.D.) 2001 5. A febrile 23 year old college coed presents with fatigue and difficulty with swallowing, Physical exam reveals exudative tonsillitis, palatal petechia, cervical lymphadenopathy, and tender hepatosplenomegaly. A CBC reveals a mild microcytic anemia, lymphocytic leukocytosis with ~ 20% of the lymphocytes exhibiting atypical features, and a mild thrombocytopenia. You would ‘expect this patient to have... . alow TIBC anormal serum ferritin an elevated total bilirubin heterophile antibodies normal serum AST and ALT titers MoOmwD> 6. A 65-year-old man with an elevated RBC count has a history of gout, frequent headaches, and pruritus after bathing. He presents with a sudden onset of abdominal pain and bloody diarrhea. Which of the following sets of laboratory data best represent this patient's hematologic disease? ‘RBC Mass [Plasma Volame ]Sa0z | Erythropoietin ‘A._| Increased | Normal Decreased | increased B._ [Increased | Normal ‘Normal | Increased C. [Increased | Tncreased ‘Normal [Low D.__[Normal [Decreased Normal [Normal Sap = oxygen saturation 7. A 28 year old woman presents with fever, scleral icterus, and crampy right upper quadrant pain, Physical exam reveals tendemess to palpation in the right upper quadrant and mild splenomegaly. A hemogram exhibits a total leukocyte count of 23,000/mm’, hemogiobin of 10.0 g/dL (12.0-16.0 WAL), a corrected reticulocyte count of 6% (0.5~1.5%), a mean corpuscular volume (MCV) of 80 uum’ (80-100 pm’), and a mean corpuscular hemoglobin concentration (MCHC) of 38% (31-36% Hb/cell). The total bilirubin is 3.2 mg/dL (0.1-1.0 mg/dL) with a direct (conjugated) fraction of 0.4 mg/dL (0.0-0.3 mg/dL). Serum alanine aminotransferase is 20 U/L (8-20 U/L). An ultrasound reveals numerous stones in the gallbladder. The common bile duct is not dilated. Based on these findings, which of the apply to this case? SELECT 3 A. Increased RBC osmotic fragility B. Calcium bilirubinate stones C. Low serum ferritin D. RBCs witha defect in spectrin E. Positive direct Coombs’ F. Decreased unconjugate bilirubin 8. A significant number of employees in a pottery factor develop abdominal colic and diarthea. In addition, a few of the employees complain of burning feet and muscle weakness in the upper and lower extremities. Based on these findings, which of the following are expected laboratory findings in these patients? SELECT 4 A Normocytic anemia Coarse basophilic stippling Low serum ferritin Increased blood Pb levels Increased RBC protoporphyrin levels Increased plasma homocysteine levels Ringed sideroblasts ommouow 14 Note: This 10. 1 1s material is copyrighted. All rights reserved. (Edward F. Goljen, MD.) 2001 'A33 year old African-American medical missionary, who recently retumed from a 2 year tour in India, is diagnosed with leprosy. After 1 week of therapy, he develops fever, chills, low back pain, wtp dark colored urine, A CBC reveals a hemoglobin of 6 g/dl. (13.5-17.5 g/4L), leukocyte count of 15,000/mm? (4500-11,000/mm), and a platelet count of 450,000/mm? (150,000-400,000/mm’) ‘A corrected reticulocyte count is 18% (0.51. 9%). A direct and indirect Coombs’ test is negative ‘Fas peripheral smear exhibits polychromasia and RBCs missing Pars of their membrane, There is a positive urine dipstick for blood. The urine sediment is reported as normal. Which of the following Statements apply to this case? SELECT 3 moODE “Heinz bodies are likely present Predominantly extravascular hemolysis Low RBC glutathione levels Patient is taking dapsone ‘Autosomal recessive disease ‘an afebrile 65 year old man presents with fatigue and substermal chem: pain with exertion that is Gelieved by resting. Physical exam reveals a blood pressure of 100/70 mm He, pulse of 110 Teint (diminished amplitude), pale conjunctiva, and harsh grade II-IV systolic murmur beats Mation into the carotid arteries. The intensity of the murmur Inowenses ‘with expiration and win The patient is lying down. A CBC exhibits, a moderately severe Tse with a mean corpuscular volume (MCV) ‘of 76 um? (80-100 pm”), hemoglobin of 7.5 g/dL (13.5-17.5 g/dL), leukocyte count of 8,000/mm* (4500-11,000/mm"), and a platelet count of 500,000/mm° (150,000—400,000/mm") The peripheral smear uncovers numerous fragmented RBCs. The ccmrected reticulocyte count is 15% (0.5~1.5%), There is @ postive Ste dipstick for blood but Rae are not present inthe sediment. Which of the following apply © this case? SELECT 4 mmoOwE Positive direct Coombs’ test Low plasma haptoglobin Schistocytes ‘Hemoglobinuria ‘Aortic regurgitation Low serum ferritin ‘An asymptomatic, normotensive 21 year old Aftican-American woman is noted to have a aevopie hematuria on an otherwise normal physical exam. The vane culture retums negative. seerrnal ultrasound is reported as normal. Her hemoglobin concentration if 12.5 g/dL (120-160 shat) and the peripheral smear is reported as normal. Basco on these findings, which of the erTowing is the next best step inthe management ofthis patient? moOw> Sickle cell preparation Reticulocyte count Renal biopsy Cystoscopy No further work-up 15 Note: This material is copyrighted. All rights reserved. (Edward F. Goljan, M.D.) 2001 12. A pregnant 21 year old African-American woman has a mild microcytic anemia with a normal RDW and slightly elevated RBC count. A Hgb electrophoresis is reported to be abnormal. You ‘suspect that the Hgb electrophoresis showed... SELECT 3 A. increased Hgb S B. decreased Hgb A C. increased Heb A D. increased Hgb F E. increased Hgb H 13. A 46 year old man with diastolic hypertension develops fever, jaundice, and a severe hemolytic anemia while taking methyldopa. The hematocrit is 15% (41-53%) and the uncorrected eticulocyte count is 24%. Spherocytes and polychromasia are present in the peripheral smear. Which of the following most likely apply to this case? SELECT 3 A. Intravascular hemolytic anemia B. _ Reticulocyte index is 4% C. Unconjugated hyperbilirubinemia D. Type Il hypersensitivity reaction E. Autoantibodies against Rh antigens 14. A 65 year old man presents with fever and chills. Physical examination reveals generalized, non- tender lymphadenopathy, hepatosplenomegaly, and scattered petechia and ecchymoses over the anterior chest. A CBC report indicates a hemoglobin of 8.2 g/dL (13.5-17.5 g/dL), leukocyte count ‘of 70,000/mm’ (4,500-11,000/mm*) with 90% mature and immature appearing lymphocytes, and a platelet count of 80,000/mm? (150,000-400,000/mm). The peripheral smear contains numerous smudge cells. A blood culture is positive for Streptococcus pneumoniae. The total serum protein concentration is 4.0 g/dL. (6.0-7.8 g/dL). A serum protein electrophoresis exhibits a flat y-globulin peak and decreased albumin, Which of the following apply to this case? SELECT 3 Leukemoid reaction secondary to sepsis Hypogammaglobulinemia Chronic lymphocytic leukemia Low leukocyte alkaline phosphatase score B cell malignancy moOOw> 15, A 4 year old boy presents with fever, epistaxis, and testicular pain. Physical exam reveals generalized, non-tender lymphadenopathy, hepatosplenomegaly, steal tendemess to percussion, and widespread petechia and ecchymoses. Both testicles are enlarged, slightly tender, and do not transilluminate. The CBC report indicates a hemoglobin of 6 g/dL (11.0-14.0 g/dL), leukocyte count of 30,000/mm? (5,500-15,500/mm’), and a platelet count of 50,000/mm* (150,000—-400,000/mm*). A bone marrow aspirate reveals sheets of cells similar to those present in the peripheral blood. Which of the following apply to this case? SELECT 3 Cells are most likely CALLA positive Cells are most likely Tat positive Acute lymphoblastic leukemia Lymphoid leukemoid reaction Leukemia derives from trilineage myeloid stem cell PoOp> 16 Note: This material is copyrighted. All rights reserved. (Edward F. Goljan, M.D.) 2001 16, A-4S year old man presents with fever, weight loss, and sweating. Physical exam reveals a Tonmotensive individual with generalized, non-tender lymphadenopathy | and_ massive hepatosplenomegaly. ‘The CBC report indicates a hemoglobin concentration of 7 ysl (150,000-400,000/mm*), leukocyte count of 150,000/mm? (4,300-11,000/mm’), and a platelet count of 650,000/mm? (150,000-400,000/mm’). The peripheral smear demonstrates neutrophils at Si sages of development (1% myeloblasts), mature and immature eosinophils and basophils, and thrombocytosis with giant platelets. A bone marrow aspirate is dry, however, the bone marrow biopsy reveals a hypercellular marrow with an increase in reticulin fibers. Imprints of the biopsy prope! f similar differential count as that observed in the peripheral blood. Which of the following Uditional laboratory test abnormalities would you expect inthis patient? SELECT 3 Low leukocyte alkaline phosphatase score Positive tartrate resistant acid phosphatase stain ‘Auer rods in myeloblasts Positive Philadelphia chromosome study Positive ber-fusion gene study 1-8;14 translocation mao Om > 17.19 year old African American woman presents with fatigue and exereise intolerance. She hhas a history of menorthagia and sporadically takes ferrous sulfate tablets. A CBC reveals & mild RSocytic anemia, a 1ow normal WBC count, a normal platelet count, and normal RBC morphology. A corrected reticulocyte count is <2%. The next most important step iS a. ‘A. serum ferritin B. Coombs’ test C. serum folate/Bi2 D. _ Hgb electrophoresis E. _ sickle cell preparation 1s, Anafebrile 80 year old man with the myelodysplastic syndrome is symptomatic with a 7 gm/dl femoglobin. He is given 3 units of packed RBCs and on the following day has a hemoglobin of 8 g/dL. His direct Coombs’ test is negative. A dipstick of urine for blood is negative. The MOST TIKELY cause for only a 1 gm/4L rise in the post-transfusion hemoglobin concentration 15. destruction of the RBCs in the bone marrow 1a microangiopathic hemolytic anemia an autoimmune hemolytic anemia destruction of the RBCs in spleen ‘a gastrointestinal bleed roOw> 19. A 22 year old college student presents with petechia, ecchymoses, epistaxis, generalized iymphadenopathy, and hepatosplenomegaly. A CBC reveals a normocytic anemia, & total WBC creer of 30,000 cells/iL, and thrombocytopenia. Abnormal “blast cells” are noted in the peripheral Setar The PT and PTT are prolonged, and the D-dimer test is positive. Which of the following apply to this case? SELECT 2 Positive tartrate resistant acid phosphatase stain ‘Therapeutic response to transretinoic acid Low leukocyte alkaline phosphatase score 1(15517) translocation Invasion of the gums moOD> 7 Note: This material is copyrighted. All rights reserved. (Edward F. Goljan, M.D.) 2001 20. 21. 22. 23. 24. Which of the following is more often associated with B-thalassemia minor than iron deficiency? SELECT 2 ‘Low MCV Increased RDW Increased serum ferritin ‘Normal to high RBC count Abnormal Hgb electrophoresis POODp ‘A 28 year old woman presents with fever, fatigue, and scleral icterus. She is not taking any prescription or over-the-counter medications. Physical exam reveals generalized painful lymphadenopathy, hepatosplenomegaly, and a facial rash in a butterfly distribution. A CBC exhibits a Hgb of 6 g/dL, a slightly increased MCV, thrombocytopenia, and slightly low WBC count. The peripheral smear demonstrates spherocytes, RBC polychromasia, and occasional nucleated RBCs. No hypersegmented neutrophils are present. The uncorrected reticulocyte count is 15%. Which of the following apply to this case? A. Bilirubin in the urine B. Positive direct Coombs’ test C. Unconjugated hyperbilirubinemia D. _ Reticulocyte index < 2% E. _ Positive serum antinuclear antibody test ‘You would expect a mature RBC to... SELECT 2 utilize lactate dehydrogenase utilize glucose 6-phosphatase convert glucose into glycogen utilize pyruvate dehydrogenase have a net gain of (2) ATP and (0) NADH roOD> ‘An afebrile patient with SLE has a mild normocytic anemia with an elevated leukocyte count exhibiting neutrophilic leukocytosis, lymphopenia, eosinopenia, and a normal platelet count. There is no left shift or toxic granulation present in the smear. Her stool guaiac is positive. The hematologic findings are MOST CONSISTENT with... acute leukemia a viral infection abacterial infection gram negative sepsis corticosteroid therapy ROOD ‘Which of the following hemolytic anemias are primarily due to extravascular removal of RBCs? SELECT 2 ‘A. Microangiopathic hemolytic anemia B. Warm autoimmune hemolytic anemia C. Congenital spherocytosis D. GOPD deficiency E, _ Paroxysmal nocturnal hemoglobinuria F., Cold autoimmune hemolytic anemia 18 i ‘ e € t t t ‘ ‘ ‘ t 4 i t t t t t t t ‘ t ‘Note: This material is copyrighted. All rights reserved. (Edward F. Goljan, M.D.) 2001 25. 26. 21, 28. 29, Which of the following correctly describe anemia? SELECT 2 A. Hypoxemia B. Low oxygen saturation C. Decreased oxygen content D. __ Left shifted oxygen dissociation curve E. Tissue hypoxia 'A.65 year old man presents with joint pains, palpable purpurs, and hepatosplenomegaly. There is a cadence of lymphadenopathy. A CBC exhibits a normocytic anemis, thrombocytopenia, and weatopenia. A few “blast cells” with iregular cytoplasmic borders te noted in the peripheral Trends bone marrow biopsy reveals a monomorphic infiltrate of eefis ‘with abundant cytoplasm having a “fried egg” appearance. A special stain is pending. ‘Which of the following apply to this case? SELECT 2 1(9;22) translocation Epstein-Barr virus association B cell malignancy Positive tartrate resistant acid phosphatase stain Low leukocyte alkaline phosphatase score MOO ‘An 82 year old man has a long history of a severe slightly macrocyt anemia and pancytopenia. He fequires a transfusion of packed RBCs every 2 weeks in order 10 ‘maintain his Hb concentration aid 10 gldL. His peripheral smear consistently shows a dimorphic ‘RBC population associated seth occasional myeloblasts and progranulocytes. A bone marrow aspirate reveals numerous ringed wieroblasts and a 10% myeloblast count. This patient MOST LIKELY has. ‘A, acute myelogenous leukemia B. agnogenic myeloid metaplasia C. the myelodysplastic syndrome D. _ chronic myelogenous leukemia E. _aneutrophilic leukemoid reaction |A°53 year old woman with an abnormal mammogram of te lft breast palpable left axillary lymph sides and low back pain is noted to have a mild normocytic anemia, ‘thrombocytosis, a slightly nodes STWBC. count, and a peripheral smear demonstrating nucleated RBCs, and occasional progranulocytes and myelocytes. The findings are consistent with a... SELECT 2 ‘A. chronic myelogenous leukemia B. acute myelogenous leukemia C. _ marrow infiltrative disease D. _ leukoerythroblastic smear E. _ leukemoid reaction osinophilia is commonly associated with... SELECT 4 A. pinworms B. bronchial asthma C. malaria D. amebiasis E, Addison's disease F. poisonivy G. _ penicillin skin rash H. _ strongyloidiasis 19 ‘ote: his material is copyrighted. All rights reserved. (Edward F. Goljan, M.D.) 2001 30. Toxic granulation, left shift, and neutrophilic leukocytosis are expected in...SELECT 4 acute appendicitis Crohn's disease acute diverticulitis acute cholecystitis acute myocardial infarction rheumatoid arthritis mmoOwD 31. Pure RBC aplasia is most commonly associated with... SELECT 2 A. parvovirus infection B. chronic myelogenous leukemia C. thymoma D. sickle cell trait E, autoimmune hemolytic anemia 32, A2yrold child has a microcytic anemia. Which of the following is the first step in the work-up of, the patient? ‘A. Serum ferritin B. Stool guaiac C. Bone marrow D. Direct Coombs test E. _Hgb electrophoresis, 33. 55 yr old man has a microcytic anemia. Which of the following is the first step in the work-up of the patient? A. Serum ferritin B. Stool guaiac C. Bone marrow D. Direct Coombs test E, _Hgb electrophoresis In lead poisoning, the encephalopathy is directly attributed to... A. an increase in RBC protoporphyrin B. an increase in 5-aminolevulinic acid C. Pb depositing in the brain D. _analteration in Starling's forces E. _ tissue hypoxia secondary to anemia 35. Ina patient with By, deficiency who is being treated with pharmacologic doses of folate, which of the following will be corrected? Megaloblastic anemia Glossitis Pancytopenia Vibratory sensation in the lower extremities Proprioception Dementia mmpomD> 20 - Note: This material is copyrighted. All rights reserved. (Edward F. Goljan, M.D.) 2001 36. Newboms with sickle cell disease do not have hemolytic or vasoocclusive crises at birth since... SELECT 2 ‘A. the high concentration of Hgb F inhibits sickling B. _ levels of Hgb S are not high enough to induce sickling C.Hgb A inhibits sickling D. splenic macrophages are of insufficient number to remove sickle cells E. the spleen traps the sickle cells 37. A 19 yr old African American man with sickle cell disease develops bone pain in the fernur. A radionuclide bone scan reveals a lytic lesion in the metaphysis of the femur. The pathogen most likely responsible for the patient's bone disorder is. A. Staphylococcus aureus B. Streptococcus pneumoniae C. Salmonella paratyphi D. Pseudomonas aeruginosa E, _ Hemophilus influenzae 38, Ina patient with sickle cell trait, you would expect the MSTH endonuclease studies to reveal 1.35 kb segment | 1.15kb fragment | 0.2 kb fragment A__| @) segments ‘none, one B. (1) segment, (i) fragment () fragment c none (2) fragment (2) fragments Lymphoproliferative questions Items 1-2 'A 65 year old woman presents with fatigue and pain in her lower back and ribs. She states that her turine flow has also decreased dramatically in the last few days. Physical examination reveals sternal and vertebral percussion tendemess and bilateral conjunctival pallor. A CBC exhibits extensive rouleaux, a hemoglobin of 7.5 g/dL (12.0-160 g/dL), a total leukocyte count of 4,300/mm? (4500-11,000/mm’), and a platelet count of 125,000/mm? (150,000—400,000/mm'). A urinalysis exhibits a 2+ dipstick for protein and 4+ precipitation using sulfosalicylic acid (SSA). ‘Renal tubular casts are noted in the sediment. A chest x-ray reveals generalized osteopenia in the ribs and vertebra as well as multiple lytic lesions in the ribs. Based on the patient's history and preliminary laboratory findings, which of the following additional abnormalities would you expect? SELECT 3 Hypercalcemia ‘Normal erythrocyte sedimentation rate IgM monoclonal spike in serum Increased light chains in the urine Malignant plasma cells in a bone marrow aspirate noOw> ‘What additional tests should be performed to confirm the diagnosis? SELECT 2 Bone marrow aspirate Renal biopsy Radionuclide bone scan Intravenous pyelogram (IVP) Serum/urine immunoelectrophoresis rmoOD> 2 Note: This material is copyrighted. All rights reserved. (Edward F. Goljan, M.D.) 2001 Items 3-7 Histocytosis X Sezary syndrome Hodgkin’s disease Burkitt’s lymphoma Immunoblastic lymphoma Waldenstrom’s macroglobulinemia mmpOMD> Each set of options relating to lymphohistiocytic disorders is followed by several numbered items. For each numbered item, select the ONE lettered option that is MOST CLOSELY associated with it. Each lettered option may be selected once, more than once, or not at all. 3. A9 year old girl presents with colicky pain secondary to entrapment of small bowel by enlarged para-aortic lymph nodes. A section of lymph node removed at laparotomy reveals a diffuse neoplastic infiltrate of small, round lymphocytes with a "starry sky" appearance. A 68 year old man presents with multiple plaque-like lesions on his skin, generalized lymphadenopathy, and hepatosplenomegaly. A biopsy reveals atypical lymphocytes infiltrating the epidermis. Neoplastic CD, positive lymphocytes with prominent nuclear clefts are noted in his peripheral blood. AA year old child presents with exophthalmos, polyuria, and multiple lytic lesions in the skull. A ‘bone marrow aspirate reveals an infiltrate of neoplastic cells that are CDI positive. A 60 year old man has a normocytic anemia, an elevated erythrocyte sedimentation rate, generalized lymphadenopathy, hepatosplenomegaly, hyperviscosity syndrome and an abnormal finding on a serum protein electrophoresis. 7 A 28 yr old woman has an anterior mediastinal mass and non-tender lymphadenopathy in the right supraclavicular node Restrictive cardiomyopathy, macroglossia, and nephrotic syndrome are all associated with a disorder characterized by... neoplastic histiocytes in a bone marrow aspirate neoplastic plasma cells in a bone marrow aspirate neoplastic lymphocytes with a t(8;14) translocation Iymphoplasmacytoid cells and a monoclonal increase in IgM material exhibiting apple green birefringence under polarized light POOm> 22 Note: This material is copyrighted. All rights reserved. (Edward F. Goljan, M.D.) 2001 Coagulation questions 1. A 22-year-old African American woman with no previous bleeding history develops persistent bleeding of her gums and tooth socket following dental surgery. Additional history reveals problems related to heavy menses that significantly resolve when she is taking birth control pills ‘and recur to the same level of severity when she discontinues the medication. Hematologic studies reveal the following: partial thromboplastin time (PTT) 55 sec (28-40 sec), prothrombin time (PT) 12 see (11-15 sec), factor VII: coagulant 30% (50-150%), factor VII: antigen 40% (60-150%), bleeding time 15 min (2-7 min), platelet count 300,000 mm’ (150,000-400,000 mm”), hemoglobin 13.0 gidL (12.0-16.0 g/dL). Which of the following laboratory tests is most useful in confirming the cause of her bleeding? A. Leukocyte count B. _ Ristocetin cofactor assay C. Hemoglobin electrophoresis D. _ Sickle cell preparation E. Serum ferritin 2. A 23-year-old man requires a root canal for an abscessed tooth for which he has been taking pain medication. On the day prior to the procedure, he develops a severe nosebleed, which prompts his dentist to order a few laboratory studies, which are as follows. PTT 35 sec (28-40 sec), PT 13 sec (11-15 sec), bleeding time 16 min (2-7 min), platelet count 200,000 mm? (150,000~400,000 mm’), Hgb 15.5 g/dL (13.5-175 g/dL). Which of the following hemostasis abnormalities is most likely present in this patient? A. Von Willebrand's disease B. Hemophilia A C. Factor IX deficiency D. Acquired platelet defect E. Acquired vascular defect Items 3-4 ‘A 63 year old man with urinary retention secondary to benign prostatic hyperplasia develops fever ‘and chills shortly after insertion of an indwelling catheter. Physical exam reveals warm skin and a bounding pulse. Within 24 hours, he begins oozing blood out of venipuncture sites and from his mucous membranes. Ecchymoses appear over his trunk and extremities. His urine output decreases to <400 mLiday. Laboratory studies reveal the following: Hgb 10 g/dL. (13.5-17.5 g/dL), WBC count 2,000/mm? (4,500-11,000/mm:), platelet count 140,000/mm? (150,000-400,000/mm’), PIT 42 sec (28-40 sec), PT 18 sec (11-15 sec), plasma fibrinogen 150 mg/dL (200-400 mg/dL), fibrin(ogen) degradation products >10 ig/ml. (<10 g/mL), D-dimers positive (negative), blood cultures pending, serum BUN 80 mg/dL (7-18 mg/dL), serum creatinine 8 mg/dL (0.61.2 mg/dL) 3. Which of the following apply to this case? SELECT 4 ‘Thrombotic thrombocytopenic purpura Disseminated intravascular coagulation ‘Consumption of coagulation factors Prerenal azotemia ‘Secondary fibrinolysis Decreased production of platelets Increased total peripheral resistance Endotoxic shock ROMMOOWD Note: This material is copyrighted. All rights reserved. (Edward F. Goljan, M.D.) 2001 4. Which of the following is the most effective treatment for this patient? Fresh frozen plasma Packed red blood cells Platelet concentrates Low dose heparin Antibiotics Cryoprecipitate mmoOw> 5. Which of the following laboratory test results are more often associated with classical von Willebrand's disease rather than mild hemophilia A? SELECT 3 A. Nonmal prothrombin time B. Low factor VIll:coagulant activity C. Abnormal ristocetin cofactor assay D. Prolonged partial thromboplastin time E, Prolonged bleeding time F, Low VIII: antigen G. Normal platelet count H. Response to desmopressin acetate Items 6-12 Platelet count Bleeding time PTT PT A. Decreased Prolonged. = Normal Normal B, Decreased Prolonged Prolonged ~——Prolonged C. Normal Prolonged Prolonged Normal D. Normal Normal Prolonged Normal E. Normal Normal Prolonged Prolonged AB. Normal Normal Normal Prolonged AC. Normal Prolonged Normal ‘Normal Each set of options relating to hemostasis studies is followed by several numbered items. For each numbered item, select the ONE lettered option that is most closely associated with it. Each lettered option may be selected once, more than once, or not at all. 6. A4yyrold child has eaten rat poison and is hemorrhaging 7. _AS4 yr old man is on heparin to prevent deep venous thrombosis 8 A 28 yr old man has been envenomated by a rattlesnake and is bleeding from all orifices and all needle stick sites 9. A child has eaten raw hamburgers and now has a hemolytic anemia and renal failure 10. Aman has a family history of a bleeding disorder which began with his mother's father 11. An afebrile 8 year old boy presents with epistaxis 1 week after an upper respiratory infection. Physical examination reveals scattered petechia and ecchymoses over his trunk. There is no lymphadenopathy or hepatosplenomegaly. A stool for occult blood is negative. His CBC exhibits a hemoglobin of 13 g/dL. (12.0-15.0 g/dL), leukocyte count of 8,500/mm? (4,500-13,500/mm’) with anormal differential count, and a platelet count of 10,000/mm’ (150,000-400,000/mm’). 24 ‘Note: This material is copyrighted. All rights reserved. (Edward F. Goljan, M.D.) 2001 12, A.38 year old woman presents with fever, mental status alterations, and epistaxis. Physical A amuiation reveals retinal hemorthages, widespread petechia and ecchymoses, and a postive stoo) Fra ovalt blood. There is no lymphadenopathy or hepatosplenomegaly. The CBC report indicates the presence of a normal hemoglobin and leukocyte count. The corrected reticulocyte Cows 12%, ‘The peripheral smear exhibits numerous fragmented RBCs, polychromasia, and @ reduced number of platelets. A bone marrow biopsy contains vascular channels containing platelet thrombi. There is «positive urine dipstick for blood and numerous RBCs aze present in the sediment ‘The serum tiood uree nitrogen is 40 mg/dL (7-18 mg/dL) and the serum creatinine is 4 mg/dL (0.6-1.2 mg/dL). 13, Am 18-year-old smoking male presents with deep venous thrombosis (DVT) involving the right treo He has a family history of recurrent DVTS and pulmonary emboli in his mother and veteral grandfather. His PTT and PT are normal prior to receiving a standard dose of hepatp metrenowsly, The PTT remains normal 1 hour after infusing heparin. The pathogenesis of his hemostasis abnormality is most likely related to... ‘antiphospholipid antibodies a thrombohemorthagic disorder a deficiency of antithrombin IT a deficiency of a vitamin K-dependent factor antibodies directed against heparin moOOw> 14, The pathogenesis of hemorthagie skin necrosis associated with warfarin therapy is most closely associated with... ‘A. antibodies directed against warfarin B. a drug hypersensitivity reaction CC. immune vasculitis secondary to warfarin D. _ protein C deficiency in the patient E. antithrombin III deficiency in the patient Blood Bank questions 1. A665 year old man in an intensive care unit is recovering from surgery for a ruptured abdominal A ic aneurysm, The patient required 12 units of packed red blood cells prior to surgery in order te sotniae his blood pressure. On the Sth postoperative day, he develops fever, scleral icterus, and sae pack pain. Physical exam reveals scattered rhonchi throughout both lung fields but no areas of toe otidaron. A urine sample taken from his indwelling urinary catheter exhibits pyuria, Cultures sonsted E.coli, A CBC reveals a 3 g/L drop in his hemoglobin concentration when compared to hus postoperative levels. A direct Coombs’ testis positive. No hemoglobin is noted im plasm The rae biliabin is 4 mp/aL (0.1-1.0 mg/dL) with a direct (conjugated) bilirubin of 04 me/dl {010-03 mg/dL). The serum alanine aminotransferase concentration is 20 U/L (8-20 U/L) Which of the following apply to the patient's clinical condition? SELECT 3 Febrile transfusion reaction Posttransfusion hepatitis Delayed hemolytic transfusion reaction Type Il hypersensitivity reaction Conjugated type of hyperbilirubinemia Extravascular hemolysis, mmOOwP 25 Note: This material is copyrighted. All rights reserved. (Edward F. Goljan, M.D.) 2001 ‘A blood group O Rh negative woman with a negative antibody screen delivers a blood group A, Rh positive baby. The baby is mildly anemic and develops an unconjugated hyperbilirubinemia in the first 24 hours. A direct Coombs' of cord blood returns weakly positive. ‘Spherocytes are noted in the peripheral blood smear. The baby's stool is grossly bloody. An Apt test is performed on the stool sample and reveals adult hemoglobin. The ‘mother's antibody screen is negative. Which of the following best describe the pathogenesis of the baby's anemia? SELECT 4 ABO incompatibility Rh incompatibility ‘Type II hypersensitivity reaction ‘Mother is not a candidate for Rh immune globulin ‘ABO incompatibility protects against Rh sensitization Positive direct Coombs is due to anti-D coating babies RBCs ‘Newbom has a GI bleed AmMoOw> ‘A group O, Rh negative 32 year old woman develops fever of 103° F while receiving a unit of O, Rh negative packed red blood cells after a hysterectomy. A transfusion reaction workup on the patient reveals the following: ‘Antibody screen negative Direct Coombs’ negative Plasma normal color Urinalysis dipstick negative for blood Pretransfusion hemoglobin 7 g/dL (12.0-16.0 g/dL) Post-transfusion hemoglobin 7.5 g/dL (12.0-16.0 g/dL) Which of the following apply to this case? SELECT 2 Donor blood contamination with Yersinia enterocolitica ‘Type I hypersensitivity reaction Patient should receive leukocyte poor blood for further transfusions Hemolytic transfusion reaction due to antibody destruction of donor RBCs Patient anti-HLA antibodies are directed against donor leukocytes moOw> ‘An 82 yr old woman with blood group A inadvertently receives blood group B blood. He does not develop a hemolytic transfusion reaction. This is most likely due to. ‘A. absence of isohemagglutinins with old age a defect in cellular immunity Bruton's agammaglobulinemia antithrombin III deficiency absent anti-A IgM titers in the donor unit Which of the following is more often associated with Rh hemolytic disease of the newborn due to anti-D antibodies rather than ABO incompatibility? SELECT 2 ‘A. Norisk for hemolytic disease of the newborn during the first pregnancy B. Positive direct Coombs’ on the babies cord RBCs CC. Spherocytes in the newbomn’s peripheral blood D. _ Severe anemia requiring blood transfusion E. Unconjugated hyperbilirubinemia F. Type I hypersensitivity reaction 26 ae ee ee ee Note: This material is copyrighted. All rights reserved. (Edward F. Goljan, M.D.) 2001 'A blood group O, Rh negative 65 old year old man with known divertculosis presents with ‘assive lower GI bleed (hematochezia). He has an estimated blood loss of greater than 600 ml. Over the last few hours. An intravenous line with 0.9% normal saline is in place, while blood is being crossmatched in the blood bank. His Hgb is 6 g/4L. Physical exam reveals cold, clammy skin, 2 blood pressure of 70/40 mm He, and a weak pulse with arate of 120 bpm. Owing to a shortage of ©, Rh negative blood only 1 of the 5 units of packed RBCs are O, Rh negative, while the remaining ‘funits are O, Rh positive. The patient has no history of a previous transfusion or exposure to blood products. Midway through the second unit of blood, which is the first of the four units of O. Rh Positive blood, he develops wheezing, dyspnea, and swelling of his face. The transfusion is topped, a subcutaneous injection of aqueous epinephrine at a 1:1000 dilution is given to the patient, and a transfusion reaction work-up is ordered. The transfusion reaction work-up on 2 post transfusion specimen of patient blood is as follows: Patient temperature: 100.0 °F Patient blood pressure: 60/40 mm Hg Patient pulse: 130 bpm Patient plasma: clear Patient antibody screen: negative Patient direct Coombs’ negative Patient urine: negative dipstick for blood “The pathogenesis of this patient's transfusion reaction is most closely related to... SELECT 2 anti-D antibodies destroying donor D antigen positive RBCs patient anti-HLA antibodies reacting against donor leukocytes ‘a patient IgE-mediated reaction against a donor allergen a type I hypersensitivity reaction an intravascular hemolytic anemia moow> ‘A blood group ©, Rh negative woman with a negative antibody screen and no previous cdministration of Rh immune globulin during her pregnancy delivers a blood group B, Rh positive baby. ‘The baby develops unconjugated hyperbilirubinemia a few hours after birth. The pathogenesis of the baby's jaundice is most closely related to... ‘A. maternal anti-D antibodies destroying the babies Rh positive RBCs intravascular hemolysis of fetal RBCs by anti-B IgM antibodies intravascular hemolysis of fetal RBCs by anti-A,B IgG antibodies extravascular hemolysis of fetal RBCs by anti-B IgG antibodies extravascular hemolysis of fetal RBCs by anti-A,B IgG antibodies moop ‘major crossmatch... SELECT 3 prevents post-transfusion hepatitis due to hepatitis C does not guarantee survival of the infused donor RBCs rules out the possibility of a febrile transfusion reaction does not prevent patient antibodies developing against donor RBC antigens ‘prevents antibodies in the donor from destroying patient RBCs detects the presence of patient antibodies against donor RBC antigens mmo OWE > 27 Note: This material is copyrighted. All rights reserved. (Edward F. Goljan, M.D.) 2001 Items 9-12 Forward type using— Back type using— anti-A anti-B A RBCs B RBCs ‘A. positive negative negative positive B. positive positive negative negative C. negative positive positive negative D. negative —negative positive positive Each set of options relating to ABO blood group typing is followed by several numbered items. For each ‘numbered item, select the ONE lettered option that is most closely associated with it. Each lettered option may be selected once, more than once, or not at all. 9%. 10. u 12. 13. 14. 'A 45 year old man with exertional dyspnea and extreme fatigue and a long history of duodenal Ulcer disease is noted to have dark black, tary stools. Physical exam reveals pale conjunctiva and a wwide pulse pressure. Laboratory studies reveal a 6 g/dL hemoglobin and a low MCV. Donor blood for transfusion of this patient based on the type of peptic ulcer disease he has would most likely have which of the above forward and back type results in the blood bank” 'A.32 year old woman and her husband have 10 children. Her husband's blood group is A. All of their children are either A, B, or AB. In order to explain the blood groups in her children, you ‘would expect the woman to have which of the above forward and back type results on a sample of her blood? A 49 year old has a gastric adenocarcinoma. Based on his diagnosis, which of the above blood szoup types is best for transfusing the patient? {A patient with this blood group could develop a hemolytic transfusion reaction after receiving AB blood or A blood ‘A phlebotomist inadvertently sticks himself with a needle after drawing blood from a patient with ‘AIDS. Which of the following infections is the phlebotomist at most risk for contracting? A. HIV B. Hepatitis A C. Hepatitis B D. Hepatitis C E. Syphilis > phlebotomist inadvertently sticks himself with a needle after drawing blood from a patient. ‘ch of the following infections is the phlebotomist at most risk for contracting? HIV Hepatitis A Hepatitis B Hepatitis C Syphilis roomEs 28 RAAB E MAMA AM Note: This material is copyrighted. All rights reserved. (Edward F. Goljan, M.D.) 2001 15. Which of the following is the most common antibody encountered in clinical practice? A. Anti-HAV-IgG B. Anti-HBs C. Anti HCV-IgG D. Anti-CMV E, _ Heterophile antibodies 16. A ppatient who has chronic hepatitis and has been transfused in the past most likely has antibodies Cardiovascular questions 1. A 72-year-old man presents with a sudden onset of left flank pain. In the emergency room, the patient is hypotensive. A pulsatile mass is palpated in the abdomen. Which of the following is MOST responsible for the pathogenesis of this patient's condition? A. Atherosclerosis B. A defect in fibrillin CC. Adefect in collagen D. _ Long-standing hypertension E. Immune destruction of elastic tissue 2. A42 year old man with a history of cardiac death in his family presents with a sudden onset of severe, retrosternal chest pain with radiation of the pain into his back. His left pulse is absent. A high pitched diastolic blowing murmur that increases with expiration is heard immediately after Sx ‘There is widening of the aortic knob on a chest x-ray. The mechanism for this patient's condition is MOST CLOSELY related to... ‘A. atherosclerosis B. a defect in fibrillin C. adefect in collagen D. an infectious process E. anacute myocardial infarction 3. A 26-year-old woman presents with a history of chest palpitations particularly when anxious. Physical exam reveals a mid-systolic ejection click followed by a murmur. The click and murmur move closer to S, when the patient is standing and closer to S; when the lying down. The mechanism for this patient’s valvular disorder is MOST CLOSELY related to.. ‘A. adefect in fibrillin B. an infectious process C. immunologic damage D. _myxomatous degeneration E. _ a defect in collagen synthesis 29 Note: This material is copyrighted. All rights reserved. (Edward F. Goljan, M.D.) 2001 4, A28-year-old patient has a family history of sudden cardiac death at a young age. The patient has a systolic ejection murmur that decreases in intensity when the patient is lying down and increases in intensity when standing up. An echocardiogram reveals abnormal movement of the anterior mitral valve leaflet against an asymmetrically thickened interventricular septum. The patient MOST LIKELY hus... mitral valve prolapse hypertrophic cardiomyopathy a congenital bicuspid aortic valve a cardiac myxoma of the left atrium infective endocarditis involving the mitral valve POOm> 5, A 65-year-old man on the Sth day of hospitalization for an acute anterior myocardial infarction has recurrence of chest pain and an increase in both CK-MB and troponin-1.The patient MOST LIKELY has... ‘A. papillary muscle dysfunction B. aright ventricular infarct C. a ventricular aneurysm D. a myocardial rupture E. reinfarction 6. — Which of the following is present in BOTH left and right heart failure? A. Ssheart sound B. Pillow orthopnea C. Pulmonary edema D. Neck vein distention E. Dependent pitting edema 7. You would expect a patient with an atrial septal defect to have which of the following oxygen saturation (SaO:) findings obtained by cardiac catheterization? [ ‘Normal | Patient | Patient | Patient | Patient | Patient | | $20;% | A B c D neues Rightatium | 75 75 80 73_| 15 80 Right ventricle | 75 80 80 75 | 75 80 Pulmonary art_|_75 80 80 30__|_75 80 {Pulmonary vein [95 95 95 95_[ 95 95 {Left ventricle 95. 95 95 95 80 80 [Aorta 95 95 95 95 | 80 80 ‘SaO; = oxygen saturation 8. A.30 year old man with a viral myocarditis who develops hypotension, neck vein distention, a drop in blood pressure on inspiration, and muffled heart sounds most likely has... hypertrophic cardiomyopathy constrictive pericarditis hypovolemic shock a pericardial effusion a dissecting aortic aneurysm PUOmP 30 ‘Note: This material is copyrighted. All rights reserved. (Edward F. Goljan, M.D.) 2001 Items 9-11 A. Aortic stenosis B. Aortic regurgitation C. Mitral stenosis D. Mitral regurgitation E. Tricuspid regurgitation 9, ATO year old man with diminished pulses and a history of angina and syncope with exercise has an ejection type murmur radiating into the carotid arteries 10, _A.58 year old man with left heart failure has an $3 and $4 heart sound and a pansystolic murmur located at the apex that increases with expiration 11.29 year old intravenous drug abuser has fever, a giant c-v wave, and a pansystolic murmur along the left sternal border that increases with inspiration 12. Which of the following types of hepatitis is associated with a vasculitis characterized by p-ANCA antibodies? A. Hepatitis A B. Hepatitis B C. Hepatitis © D. Hepatitis D E, Hepatitis E 13, AT year old boy presents with a low-grade fever, arthralgias, colicky abdominal pain, and a palpable purpuric rash limited to the lower extremities. Laboratory studies reveal a guaiae-posiit® Pool, a urinalysis with red blood cell (RBC) casts, hematuria, and mild proteinuria, and a CBC with a normal Hb, Het, and platelet count. Which of the following is the most likely diagnosis? ‘A. Idiopathic thrombocytopenia purpura (ITP) B. _ Systemic lupus erythematosus (SLE) CC. Poststreptococcal glomerulonephritis D. Rocky Mountain spotted fever E. — Henoch-Schonlein vasculitis Respiratory questions 1, In which of the following clinical scenarios involving patients with lung disease would you expect pulmonary function studies to exhibit decreased compliance, increased elasticity, and an increased FEViseeFVC ratio? ‘A. 6 year old child with recurrent respiratory infections and steatorrhes B. 28 year old non-smoking male with bilateral lower lobe emphysema C56 year old smoker with productive cough, dyspnea, and cyanosis D. 10 year old girl with ‘bronchial asthma requiring systemic steroids E. 39 year old dyspneic African American with bilateral hilar nodes 31 Note: This material is copyrighted. All rights reserved. (Edward F. Goljan, M.D.) 2001 2. A 45-year old woman 24 hours post-cholecystectomy develops fever and dyspnea. Physical exam reveals decreased percussion, increased tactile fremitus, and decreased breath sounds in the right lower lobe. The diaphragm is elevated and there is inspiratory lag on the right side. The patient MOST LIKELY has... A. atelectasis B. a lung abscess C. _bronchopneumonia D. a pulmonary infarction E. a spontaneous pneumothorax 3. An afebrile 23-year-old man develops a sudden onset of lefi-sided, stabbing chest pain with dyspnea. Physical exam of the left chest reveals hyperresonance to percussion, deviation of the ‘trachea to the left, elevation of the diaphragm, decreased tactile fremitus, and decreased breath sounds. The MOST LIKELY diagnosis i... A. pleural effusion B. _ bronchopneumonia C. tension pneumothorax D. a pulmonary infarction E. spontaneous pneumothorax 4. Anewborm child develops dyspnea, tachypnea, intercostal muscle retractions, and cyanosis 4 hours after birth. The mother developed gestational diabetes mellitus and was in poor glycemic control throughout the pregnancy. A chest x-ray reveals a “ground glass” appearance in both lungs. The Primary mechanism for this patient’s respiratory problem is... aspiration of amniotic fluid group B streptococcus pneumonia decreased production of surfactant Chlamydia trachomatis pneumonia heart failure from congenital heart disease PoOuD 5. Which of the following describes a pneumonia due to Mycoplasma pneumoniae rather than Streptococcus pneumoniae? A. High fever B. Insidious onset C. Productive cough D. Increased tactile fremitus E. _Neutrophilic leukocytosis 6. A 58-year-old smoker presents with weight loss and cough. Physical exam reveals a mild lid lag on the left and a pinpoint pupil, scattered sibilant rhonchi throughout all lung fields that clear with coughing, and an increased anteroposterior diameter. Based on these findings, you suspect the patient has... ‘A. a Pancoast tumor B. a thoracic outlet syndrome C. the superior vena caval syndrome D. _ obstructive lung disease without primary cancer E. obstructive lung disease with metastatic cancer from another primary site 32 Note: This material is copyrighted. All rights reserved. (Edward F, Goljan, M.D.) 2001 7. A.65 year old man with urinary retention secondary to prostatic hyperplasia, develops spiking fever, and tachypnea. Physical exam reveals intercostal muscle retractions and bilateral inspiratory crackles, A chest x-ray exhibits bilateral interstitial and alveolar infiltrates. ABGs demonstrate severe hypoxemia. You expect the blood culture reveals... ‘A. gram positive diplococci B. gram negative diplococci C. gram positive cocci D. _gramnegative rods E. gram positive rods 8. Inspiratory stridor is commonly associated with... a respiratory syncytial virus infection a parainfluenza virus infection aspirin-induced asthma rhinovirus infections choanal atresia MOODS 9. Chlamydia trachomatis and the respiratory syncytial virus are BOTH commonly associated with... ‘A. an interstitial type of pneumonia B. _ laryngotracheobronchitis (croup) CC. the respiratory distress syndrome D. typical community-acquired pneumonia E, _hospital-acquired (nosocomial) pneumonia 10. Which of the following is more often associated with Klebsiella pneumoniae than Pseudomonas aeruginosa? ‘A. Upper lobe cavitation B. Green-colored sputum €. Association with cystic fibrosis D. Association with respirators E. Productive cough 11. Ina 30 year old man who lives in Tennessee, you would expect a calcified solitary coin lesion in the lung to represent... A. a foreign body B. anold granuloma CC. metastatic cancer D. _aprimary lung cancer E, bronchial hamartoma 12, A'55-year-old non-smoking coal worker has arthritis and nodular lesions in the lungs. His PPD skin testis negative. You suspect the patient has... systemic lupus erythematosus Caplan's syndrome ‘metastatic lung disease primary lung cancer miliary tuberculosis, moawP 33 Note: This material is copyrighted. All rights reserved. (Edward F. Goljan, M.D.) 2001 13. 14, 15. 16. 17, In a 62 year old man who has been a roofer for 25 years and a smoker for 10 years, which of the following cancers would he be most likely prone to developing? A. Pleural mesothelioma B. Primary lung cancer C. Laryngeal carcinoma D. Oral cancer E. Pancreatic cancer A bridge painter in Brooklyn, New York develops a pulmonary infiltrate. Which of the following groups of pathogens are on your differential list? Histoplasma capsulatum and Cryptococcus neoformans Cryptococcus neoformans and Coccidioides immitis C. _ Blastomyces dermatitidis and Histoplasma capsulatum D. Aspergillus fumigatus and Coccidioides immitis E, Pneumocystis carinii and Aspergillus fumigatus p> Which of the following is a hypersensitivity pneumonitis that primarily occurs in textile workers? A. Silo filler’s disease B. Bagassosis C. Farmer's ng D. _ Byssinosis E, Sarcoidosis Which of the following is a hypersensitivity pneumonitis that commonly occurs in farmers who enter a closed room with fermenting com? A. Silo fille's disease B. _Bagassosis, C. Farmer's lung - D. — Byssinosis E, Sarcoidosis If a patient is breathing room air (21% oxygen), the arterial PCO; is 80 mm Hg, and the arterial PaO) is 40 mm Hg, what is the Alveolar-arterial (A-a) gradient of the patient? A 10 BOIS Cc 20 D. 25 E30 34 Note: This material is copyrighted. All rights reserved. (Edward F. Goljan, M.D.) 2001 Gastrointestinal questions Items 1-6 A. Vibrio cholera B. Bacillus cereus C. Shigella sonnei D. Salmonella typhi E. Campylobacter jejuni F. Yersinia enterocolitica G. Staphylococcus aureus H. _ Enterotoxigenic E. coli 1. Anafebrile 22 year old man and several other members of his family developed severe vonitg ‘without diarrhea ~1-6 hours after eating potato salad at a picnic. They all recovered uneventfully 12-24 hours later. 2, A.23 year old man developed explosive, watery diarthea with blood, leukocytes, and mucus ~3 days after eating chicken that was improperly cooked. Comma to S-shaped organisms sre noted in the fecal smear of stool along with RBCs and leukocytes. 3, A febrile 10 year old child presents with severe right lower quadrant pain that is interpreted by the ‘Trending physician as acute appendicitis. At laparotomy, the surgeon notes that the appendix is secmal. However, the mesenteric Iymph nodes are markedly enlarged and have focal areas of microabscess formation on cut section. 4, A-29 year old man develops watery diarrhea and volume depletion shortly after visiting the Gulf Coast states 5, Ona trip outside of the county, 2 man develops a high fever associated with bradycardia, absolute neutropenia, and splenomegaly. A blood culture is positive for a gram negative organism. 6. 23 yr old man develops vomiting and diarthea after eating refried riee and tacos. Gram positive rods are noted in the stool. tems 7-8 A. Yersinia enterocolitica B. Staphylococcus aureus C._ Enterotoxigenic £. coli D. Chptosporidium parvum E. Mycobacterium avium-intracellulare 7, A 28 year old man with AIDS presents with chronic, recurrent, profuse, nonbloody, watery ‘farhea, An Entero-Test (string test) reveals oocysts that are partially acid-fast positive, &. _A2S year old medical student during Spring break outside the country develops fever, vomiting: A Normal eramps, and watery diarrhea ~14 hours after eating a few tacos purchased from a stest vendor. He recovers uneventfully in 48-72 hours. 35 Note: This material is copyrighted. All rights reserved. (Edward F. Goljan, M.D.) 2001 9 Odynophagia in a HIV-positive 28 year old man with white plaque-like material on his tongue and buccal mucosa MOST LIKELY has an AIDS-defining lesion caused by... A. Epstein-Barr virus B. Candida albicans C. — Kaposi’s sarcoma D. cytomegalovirus E, Herpes simplex 10. An afebrile 25-year-old medical student presents with intermittent complaints of left and right lower quadrant abdominal pain and distention associated with alternating periods of mucoid diarrhea and constipation. He states that stooling relieves the pain. A flexible sigmoidoscopy and ‘stool guaiac exam are both normal. The Patient MOST LIKELY has. an intrinsic bowel motility disorder inflammatory bowel disease chronic appendicitis melanosis coli celiac disease mMoOD> 11. Which of the following correctly describes a gastric rather than a duodenal ulcer? A. Association with Helicobacter pylori B. Highest incidence of perforation C. Small risk for adenocarcinoma D. Pain awakens the patient at night E. Association with Zollinger-Ellison syndrome 12. The MOST COMMON location for diverticula, polyps, and cancer in the gastrointestinal tract is the... ascending colon sigmoid colon ‘esophagus stomach rectum A B. ci D. E. 13. Which of the following characterizes ulcerative colitis rather than Crohn’s disease? A. Discontinuous spread B. Toxic megacolon C. Fistula formation D. Perianal disease E. Obstruction 14. A 38-year-old Asian woman has a long history of explosive diarthea and abdominal distention after ating dairy products. The pathogenesis of this patient’s diarrhea is MOST CLOSELY related to. A. antigliadin antibodies B. activation of cyclic AMP C. intraluminal osmotically active solutes D. _ mucosal injury with increased permeability E. loss of the absorptive surface of the small bowel 36 Note: This material is copyrighted. All rights reserved. (Edward F. Goljan, M.D.) 2001 15. A 62-year old man smoker presents with weight loss, a dragging sensation in his right upper quadrant, and crampy left lower quadrant abdominal pain. He has alternating bouts of constipation and diarthea. In addition, he states that blood coats and is mixed in with his stools. He sometimes has pain with defecation. There is mild hepatomegaly. The rectal exam reveals non-thrombosed extemal hemorrhoids. The stool is guaiac positive. A complete blood cell count (CBC) reveals a mild microcytic anemia. The MOST LIKELY cause for this patient's condition is... A. angiodysplasia B. — ananal fissure C. diverticulitis D. hemorrhoids E. colon cancer 16. A.50 year old man presents with flushing of the face, watery diarrhea, weight loss, and multiple mass jesions in his liver. The primary site for the cancer that led to the above symptom complex is the... A. lung B. liver C. stomach D. appendix E. terminal ileum 17. Hematochezia in elderly patients that cannot be identified with a barium enema study is most ‘commonly due to a disorder located in the... A rectum B. stomach C. sigmoid colon D. proximal duodenum E. cecum and ascending colon 18. A febrile 58 year old man with a long history of chronic constipation complains of a steady pain in the left lower abdomen. He has had frequent attacks of pain in the same area for several months and ‘one episode of bloody stools that spontaneously resolved. Physical exam reveals rebound tendemess and a palpable mass in the left lower quadrant. A stool guaiac is negative. A CBC exhibits an absolute neutrophilic leukocytosis and left shift. A flat plate is reported to show no air under the diaphragm. The most likely diagnosis is.. A. volvulus B. colon cancer C. ischemic colitis D. acute diverticulitis E, small bowel infarction 19. A 58 yr old woman complains of epigastric pain, weight loss, vomiting of coffee ground-like material, and dark black sticky stools. Physical exam reveals epigastric pain to deep palpation and non-tender, hard left supraclavicular lymph nodes. Both ovaries are enlarged and firm on bimanual pelvic exam. Which of the following scenarios best explains the signs and symptoms in this patient? ‘A. Stomach cancer with metastasis to the ovaries B. Primary ovarian cancer with metastasis to the stomach C. Primary pancreatic cancer with metastasis to the ovaries D. Malignant lymphoma of the stomach with metastasis to the ovaries E. Primary ovarian cancer with metastasis to the supraclavicular lymph nodes 37 cre aaa Note: This material is copyrighted. All rights reserved. (Edward F. Goljan, M.D.) 2001 20. A 65 year old man with a chronic arrhythmia with an irregularly imegular beat presents with a sudden onset of severe abdominal pain associated with vomiting, abdominal distention, and bloody diarrhea. Physical exam reveals hypotension, absent bowel sounds, no rebound tendemess, and a guaiac positive, liquid stool. A complete blood cell count exhibits a white blood cell count of 35,000 cells/uL with an absolute neutrophilic leukocytosis and left shift. The serum amylase is elevated. The most likely diagnosis in this patient is... ischemic colitis hemorrhagic pancreatitis acute appendicitis with perforation acute diverticulitis with perforation hemorrhagic infarction of the small bowel moop> 21. A 42 year old woman complains of increasing pain and difficulty swallowing liquids and solids over the past several months. She has lost 15 pounds in the past three months and has occasionally experienced acute substernal pain and regurgitation of food into her mouth when lying down at night. Esophageal manometry reveals aperistalsis of the lower esophagus and increased pressure in the lower esophageal sphincter (LES). Which of the following apply to this case? SELECT 2 ‘A. Elevation in anti-centromere antibodies B. _ Involves striated muscle in upper esophagus C. Absence of ganglion cells in the LES myenteric plexus D. _ Bird's beak appearance on barium study E, Adenocarcinoma of the distal esophagus ‘Hepatobiliary/pancreas questions 1. A febrile 12 year old boy with a viral infection lapses into coma. Physical exam reveals papilledema and hepatomegaly. The serum ammonia and transaminases are elevated and the PT is prolonged. The patient MOST LIKELY has... A. viral hepatitis, B. _ Reye’s syndrome C. salicylate intoxication D. acetaminophen toxicity E. a antitrypsin deficiency 2. Which of the following serologic data best represents a patient who has recovered from hepatitis B? HBsAg HBeAg Anti-HBc-JgM Anti-HBc-lgG Anti-HBs ‘A. negative negative positive negative negative B. positive positive positive negative negative C. negative negative negative positive positive D. negative negative negative negative positive 3. An afebrile 42 year old migrant worker from the border between Texas and Mexico presents with bloody diarthea and right upper quadrant pain. The patient MOST LIKELY has... A. amebiasis B. echinococcosis C. acute cholecystitis D. ascending cholangitis E, — metastatic colon cancer 38 Note: This material is copyrighted. All rights reserved. (Edward F. Goljan, M.D.) 2001 4. A.48-year-old alcoholic with cirrhosis and chronic pancreatitis has steatorthea and a prolonged PT. The PT does not correct to normal after giving an intramuscular injection of vitamin K. You conclude that the patient MOST LIKELY has... vitamin K deficiency a circulating anticoagulant an isolated factor deficiency inadequate synthesis of coagulation factors vitamin K deficiency secondary to malabsorption moODP 5, A 45-year-old man presents with increased skin pigmentation, steatorthea, and diabetes mellitus. ‘The mechanism MOST LIKELY responsible for this constellation of findings is... amyloidosis alcoholic cirrhosis a defect in iron metabolism alpha,-antitrypsin deficiency a defect in copper metabolism moe 6. In which of the following diseases would you expect a conjugated bilirubin 50% of the total bilirubin? Gilbert's syndrome Chronic viral hepatitis Crigler-Najjar syndrome Stone in the common bile duct Extravascular hemolytic anemia rPoOp> 7. An afebrile 62 year old man with a history of alcoholism, smoking, and chronic pancreatitis presents with weight loss, a slow onset of painless jaundice, and a normocytic anemia. Physical exam reveals a palpable gallbladder and a light-colored stool. The patient MOST LIKELY has... hepatocellular carcinoma ‘carcinoma of the gallbladder primary sclerosing cholangitis, a stone in the common bile duct carcinoma of the head of pancreas PoOODP 8, A.38-year-old woman presents with chronic liver disease, greenish-brown deposits in the limbus of the eye, and choreoathetotic movements. Expected findings in this patient include... SELECT 2 high total copper levels increased percent iron saturation anti-HCV IgG antibodies low ceruloplasmin levels degenerative disease in the lenticular nuclei PoOOR> 39 Note: This material is copyrighted. All rights reserved. (Edward F. Goljan, M.D.) 2001 9. A 55-year-old woman presents with generalized pruritus, non-tender hepatomegaly, and yellow papular lesions scattered over her skin. She has marked elevation of serum alkaline phosphatase and y-glutamyltransferase, a normal total bilirubin, slightly elevated serum transaminases, and a severe hyperlipidemia. An endoscopic retrograde cholangiopancreatography (ERCP) study of the common bile duct is negative for stones. Which of the following are expected findings in this patient? SELECT 3 Hypertriglyceridemia Biliary cirthosis, Antimitochondrial antibodies Increased serum IgG levels Granulomatous inflammation room> 10. Which of the following complications is a direct by-product of portal hypertension secondary to alcoholic cirrhosis? SELECT 3 A. Ascites B. Gynecomastia C. Esophageal varices D. Hepatic encephalopathy E. _ Periumbilical caput medusae 11. A2 year old woman develops jaundice. The conjugated bilirubin fraction is > 50% of the total bilirubin and the serum alkaline phosphatase is markedly elevated. You suspect the patient is taking which of the following medications? SELECT 1 A. Isoniazid B. Allopurinol C. Tetracycline D. Acetaminophen E. Oral contraceptives 12, An afebrile high school wrestler develops jaundice, light colored stools, dark urine, and mild non- tender hepatomegaly. The urine is negative for urobilinogen and positive for bilirubin. The serum alkaline phosphatase is markedly elevated. His hepatitis serology tests and heterophile antibody test are both negative. You suspect the patient has... SELECT 1 a stone in the common bile duct been taking anabolic steroids infectious mononucleosis non A, non-B hepatitis chronic cholecystitis POOD> 13. A 42 year old Asian man with postnecrotic necrosis secondary to chronic hepatitis B has a low grade fever, weight loss, and a rapid development of ascites. A peritoneal tap reveals bloody ascitic fluid. The fluid WBC count is normal and bacteria are not present on Gram stain. The a-fetoprotein level is 500 ng/mL (normal < 6 ng/mL). The MOST LIKELY diagnosis is... SELECT 1 primary gallbladder cancer hepatocellular carcinoma metastatic liver disca spontaneous peritonitis cholangiocarcinoma moOe> Note: This material is copyrighted. All rights reserved. (Edward F. Goljan, M.D.) 2001 14, 15. 16. ‘A patient living in a Basque community in Southern Arizona presents with @ history of recurrent right upper quadrant pain. He is a sheepherder and has a dog that helps ‘him keep the sheep together. ran egsound of is liver reveals a eystic mass in the liver with caleifications in the lining of the cyst, Which ofthe following additional findings would you expect inthis patient? SELECT 1 ‘A. Hiss dog is an intermediate host B. He isa definitive host C. _Heate an infected sheep D. _ His dog ate an infected sheep ‘A weightlier develops sudden onset of abdominal pain along with hypovolemic shock. At surgery, his abdominal cavity is filled up with blood. The cause of the intraabdominal bleed is most likely associated with which of the following? SELECT 2 ‘A. Anabolic steroids B. Hepatocellular carcinoma C. Liver cell adenoma D. Cavernous hemangioma in the liver E. _ Straining at stool |A 35-year-old executive of an up and coming computer company develops a sudden onset of severe epigastric pain with radiation of the pain into the back. He describes the pain as ‘knife-like. In cairn be feels nauseous and has Vorited non-bile stained fluid on two occasions. Physical Exam ‘reveals tenderness in the epigastric area but no rebound tenderness. There is mild tender ‘hepatomegaly. A flat plate of the abdomen ‘reveals a dilated loop of small ‘bowel in the left upper quadrant An ultrasound reveals no stones in the gallbladder and a normal dismett of the common pile duct, A stool guaiac is negative. Serum AST is higher than serum ALT. What test is most indicated in this patient? SELECT 1 Upper gastrointestinal endoscopy Serum amylase and/or lipase HIDA (radionuclide) scan to R/O a cystic duct stone Hepatitis serologies ‘Surgical consult to R/O smal] bowel obstruction BPOODP Renal questions 1 ‘An 82 year old man presents with lower back pain and complaints of problems with voiding urine. ‘There is point tendemess over the lower lumbar vertebra. His bladder is percussed at the level of te ucabiheus. Which of the following tests or procedures is indicated as the first step in the management of this patient? ‘Radionuclide bone scan Prostate specific antigen Digital rectal examination Serum alkaline phosphatase Transrectal ultrasound with biopsy MOOD 4. ote: This material is copyrighted. All rights reserved. (Edward F. Goljan, M.D.) 2001 ‘A 28-year-old man with a history of removal of a right cryptorchid testis as a child is noted to have 1 painless enlargement of the remaining testicle in the left scrotal sac. The testicle is enlarged and does not transilluminate. The patient most likely has a.. A. hydrocele B. seminoma C. varicocele D. yolk sac tumor E, choriocarcinoma ‘A 66 year old man with a 45 pack year history of smoking presents with hematuria, fever, and a palpable mass in the left lower quadrant. A chest xray reveals multiple nodular masses in the lungs. These findings most strongly suggest which of the following diagnoses? Miliary TB involving the kidneys Renal cell carcinoma with lung metastasis Primary lung cancer with metastasis to the kidney Choriocarcinoma with metastasis to lungs and kidneys ‘Acute pyelonephritis with metastatic abscesses in the lung rpow> A febrile 23 year old woman presents with an acute onset of right flank pain, suprapubic discomfort, dysuria, and increased frequency of urination. The urinary sediment examination reveals clumps of leukocytes, WBC casts, occasional RBCs, and numerous motile bacteria. The mechanism of this patient's urinary condition is most closely related to. A. arenal stone B. ascending infection CC. immunocomplex disease D. drug-induced interstitial nephritis E. hematogenous spread of infection to the kidneys A 25 year old male presents with a history of hemoptysis and malaise. His blood pressure is 140/90 mm Hg. He has periorbital edema and smoky colored urine. Pertinent laboratory data include a serum urea nitrogen of 40 mg/dL. (7-18 mg/dL) and a creatinine of 4.0 mg/dL (0.6-1.2 mg/dL). Urinalysis shows 2+ protein, 3+ blood, RBCs too numerous to count, and RBC casts. A chest radiograph demonstrates opacities in both lung fields. The mechanism of this patient's lung and renal disease is most closely related to a. A. type [hypersensitivity reaction B. type I hypersensitivity reaction CC. type IIT hypersensitivity reaction D. type IV hypersensitivity reaction E, type and Ill hypersensitivity reaction 42 Note: This material is copyrighted. All rights reserved. (Edward F. Goljan, M.D.) 2001 Items 6-7 ‘A. IgA glomerulonephritis B. —Membranous glomerulonephritis, C. Focal segmental glomerulosclerosis D. _ Rapidly progressive crescentic glomerulonephritis E. Type I membranoproliferative glomerulonephritis type I 6. _A-74 year old man with a 80 pack history of cigarette smoking presents with hemoptysis and shortness of breath. Radiographs of the chest demonstrate a left hilar mass. He also has generalized anasarca with 4+ proteinuria, hypercholesterolemia, and fatty casts in the urine. 7, A.34 year old male who is a known drug seeker and heroin addict comes to the emergency department in an agitated state. He is restrained and you note multiple needle tracks on both arms. He also has severe pitting edema of the lower extremities. Urinalysis is positive for protein and shows fatty casts on urine sediment. 8. A 10 year-old-adolescent boy presents with a unilateral, tender subareolar mass in the left breast, Physical exam is otherwise unremarkable. Which of the following would you recommend for this patient? ‘A. Serum gonadotropins B. Chromosome analysis CC. Serum B-hCG D. Surgical consult E. Nottreatment 9. A 12year old child presents with dyspnea and hypertension. Physical exam reveals a palpable right Tower quadrant mass in the abdomen and scattered sibilant rhonchi.in the lungs. A chest xray reveals multiple nodular masses in the lungs. These findings most strongly suggest which of the following diagnoses? ‘Wilm's tumor ‘Neuroblastoma Carcinoid tumor Renal adenocarcinoma Dysplastic kidney disease moOm> 10. A normotensive 8 yr old boy, with a history of an upper respiratory infection 1 week ago, now presents with generalized pitting edema. The urinalysis reveals severe proteinuria, fatty casts, and val fat bodies. His condition rapidly improves with high doses of corticosteroids. Which of the following glomerular changes would you expect in this patient? Fusion of the podocytes Diffuse proliferative changes Diffuse membranous changes Subepithelial immune deposits Subendothelial immune deposits moop> 43 Note: This material is copyrighted. All rights reserved. (Edward F. Goljan, M.D.) 2001 11, A 72 yr old man presents with hypovolemic shock secondary to a ruptured abdominal aortic aneurysm. Within 24 hrs of surgery, he develops oliguria. He died 3 days later. Which of the following laboratory abnormalities is most likely present in this patient? A. White blood cell casts B. BUN(creatinine ratio > 15/1 C. Urine Osmolality > 800 mOsm/kg D. Fractional excretion of sodium > 1 E. Random urine sodium < 20 mEq/L 12, A 32 yr old man with hypertension has a systolic click followed by a murmur. He has a family history of hypertension at an early age often progressing into chronic renal failure requiring either dialysis or a renal transplant. A few of the family members have died from a stroke related to an intracerebral bleed or subarachnoid hemorrhage. The patient most likely has... A. adult polycystic kidney disease B. renal artery stenosis C. renal dysplasia D. IgA glomerulonephritis E. diabetic nephropathy 13. A32 yr old man with Wegener's granulomatosis has hematuria. A urine cytology reveals numerous RBCs and occasional atypical transitional cells. Which of the following best explains the renal problems in this patient? ‘A. Renal disease associated with Wegener's granulomatosis B. Chemotherapy agent the patient is taking to treat his disease C. Renal adenocarcinoma unrelated to his disease D. Adenovirus infection E. Lower urinary tract infection 14, A tall, obese 13-year-old boy has bilateral gynecomastia, decreased testicular volume for age, and sparse axillary and pubic hair. He has had the usual childhood infections except for mumps. A CT scan of the sella turcica is normal. A chromosome study is pending. Based on the physical findings and CT scan results, which of the following serum follicle stimulating hormone (FSH), serum luteinizing hormone (LH), and serum testosterone levels is expected in this patient? SerumFSH SerumLH Serumtestosterone Chromosomes A High Normal Normal 47 B. Normal High Low 46 Cc. High High Low 47 D. Normal Normal Normal 46 E. Low Low Low 47 Note: This material is copyrighted. All rights reserved. (Edward F. Goljan, MD.) 2001 Gynecology questions Items 1-4 Infiltrating ductal carcinoma Lobular carcinoma Fibrocystic change Intraductal papilloma Intraductal carcinoma . Paget's disease A 28-year-old woman during her breast exam is noted to have a bloody discharge. No masses are palpable. mmo ODP |A.65 year old woman with a history of breast cancer in her mothet is noted to have retraction of sn in the right upper outer quadrant when abducting her right arm during her annual clinical see An indurated 3-cm mass is palpated directly beneath the skin retraction. ‘Non-tender, firm right axillary lymph nodes present in the lower axillary chain- A 26 year old woman without any family history of breast canctt complains of pain and “jumpiness" in her breasts that progressively increases throughout her menstrual cycle and is carcred after menses is completed. Physical exam reveals a painful, ill-defined mass in the left tipper quadrant. No palpable axillary lymph nodes are present. |A.65 yr old wornan has a rash on the nipple and a subareolar mass A 58-year-old woman with a 20-year history of smoking has a history of breast cancer in her nother. Her diet is poor in fiber and rich in saturated fats. ‘Menarche occurred at 13 years of age ararrnapause began at 52 years of age. She has had 3 children. She had cervical conization at 38 years of age for severe cervical dysplasia and has since had normal cervical Pap smears. Which of the following is her greatest ‘risk factor for breast cancer? A Age B. Family history C, Smoking history D. _ Severe cervical dysplasia E. _ Low fiber, high saturated fat diet During her monthly selfbreast exam, a 19-year-old woman notes > moveable, firm, slightly tender During ne left upper outer quadrant. There is no skin retraction or nipple discharge. She is mass iy taking birth control pills. Her older sister was recently given a diagnos® of breast cancer tarrctich she opted for breast conserving therapy. You would expect a fine needle aspiration of the mass to reveal. ‘A. an infiltrating ductal cancer B. an intraductal papilloma CC. _ benign cyst fluid D. a fibroadenoma E, lobular cancer 45 Note: This material is copyrighted. All rights reserved. (Edward F. Goljan, M.D.) 2001 7. A22 year old woman, gravida 1, para 0, presents to the outpatient office for a routine prenatal visit at 34 weeks’ gestation. Her blood pressure is 150/95 mm Hg, On urine dipstick, she has 1+ glucose and 2+ albumin. Her blood pressure on the initial prenatal visit at 14 weeks’ gestation was 120/75 mm Hg. Her maternal grandfather has adult-onset diabetes. Her mother and matemal grandmother oth have chronic hypertension. Which of the following is the most likely explanation for the findings in this patient? A. Preeclampsia Molar pregnancy Abruptio placenta Gestational diabetes Primary renal disease 8. A117 year old girl presents with a sudden onset of abdominal pain. Physical examination reveals a tender mass in the left adnexa. A pregnancy test is negative. An x-ray exhibits a mass lesion of the left ovary with focal areas of calcification. Which of the following best characterizes the ovarian mass? ‘A. Benign surface-derived tumor Malignant surface-derived tumor Follicular cyst of the ovary Benign germ cell tumor Malignant germ cell tumor mpop moow 9. A 66 year old nulliparous woman presents with abdominal distention despite a poor appetite. She underwent menopause 17 years ago. A rectal exam reveals induration in the rectal pouch of Douglas. She has a fluid wave in the abdomen and a left-sided pleural effusion. A thoracentesis shows clumps of malignant cells that stain positive for CA 125.Which of the following is the most likely diagnosis? ‘A. Metastatic uterine leiomyosarcomas B. Metastatic endometrial cancer C. Metastatic ovarian cancer D. Metastatic cervical cancer E. Metastatic stomach cancer 10 A 28 year old woman, gravida 1, para 0, at 12 weeks’ gestation presents with painless vaginal bleeding. Her blood pressure is 160/95 mm Hg, there is 3+ proteinuria, and the uterus is large for gestational age. Which of the following is the most likely diagnosis? A. Gestational diabetes B. Placenta praevia C.Abruptio placenta D. Molar pregnancy E, Twin placenta 46 ML. 12. 1B. 0 BP 9 Note: This material is copyrighted. All rights reserved. (Edward F. Goljan, MD.) 2001 Place the following neoplasms of the female genital tract in decreasing incidence (morbidity) and in decreasing mortality. 1 Cervix 2. Ovary 3. Endometrium Morbidity __ Mortality A. 21,3 3,21 B. 1,3.2 2,31 G 3,21 2,13 D. 3,21 31,2 E 3,21 3,21 | ‘A 51 year old woman with a long smoking history is hospitalized for treatment of right pyelonephritis. An intravenous pyelogram shows right-sided hydronephrosis and a dilated ureter. During pelvic examination, you detect a malodorous vaginal discharge. A firm, irregular right Mineral mass extends to the pelvic side wall, The patient experiences ‘vaginal bleeding after examination. What is the most likely diagnosis? An Perinephric abscess due to acute pyelonephritis B. Endometriosis involving the right ovary Cc. Invasive cervical carcinoma D. _ Invasive endometrial adenocarcinoma E. Invasive ovarian adenocarcinoma ‘A 22-year-old moderately obese woman presents with infertility problems She has a long history av oligomenorthea. Physical exam reveals hirsutism on the face and anletion chest. Pelvic exam Cxiubits bilaterally enlarged ovaries. The following laboratory tests 2 available: serum LH srereased, serum FSH low, LH/FSH ratio >3/1, serum total testosterone slightly increased, serum ‘Per testosterone increased, serum DHEA-sulfate normal, serum prolactin normal, and the serum £5; normal, Based on these finding, you strongly suspect that the patient has. bilateral androgen-secreting tumors of the ovaries deficiency of gonadotropin-releasing hormone polycystic ovarian syndrome adrenal Cushing's syndrome ‘a gonadotropin-secreting pituitary tumor mOOwP Which ofthe following patients is at greatest rsk for developing endometrial cancer? Present Age Menarche Menopause Children Miscellaneous 45 uw - none Hypertension 54 2 st 2 Obese, took birth con- trol pills 56 2 30 none Type Il diabetes mel- titus 58 ul 3 none Obese, diabetes _mell- ites 55 u 50 none Obese, took birth con- trol pills 47 Note: This material is copyrighted. All rights reserved. (Edward F. Goljan, M.D.) 2001 15. In which of the following clinical situations, does the patient have the greatest risk for developing cervical cancer? Onset of sexual Number of Present age _activity sexual partners Miscellaneous history ‘A. 20years old 14 years old ul Obese, on birth control pills B. 19 years old 10 years old 2 On birth control pills, smoker C. 19 years old 12 years old 3 On birth control pills, obese, smoker D. 21 yearsold 10 years old 10 Intravenous drug abuser, smoker, on birth control pills E, 21 yearsold 10 years old 4 Obese, smoker, current boy- friend is an intravenous drug abuser 16. A.25 yr old woman presents with a history of severe colicky dysmenorrhea, painful stooling during ‘menses, and problems with infertility. Laparoscopy reveals scarring around both fallopian tubes, a 3 cm cyst in the right ovary, and "powder burn" appearing areas on the uterine ligaments and serosal surface of the uterus. The mechanism for this patient's constellation of problems is most likely related to... spread of endometrial carcinoma seeding from a primary ovarian tumor reverse menses through the fallopian tubes complications related to pelvic inflammatory disease ‘endometrial glands and stroma in the myometrial tissue POOm> 17. A short-statured 16-year-old girl presents with a history of primary amenorrhea. Physical exam reveals poor breast development and sparse axillary and pubic hair. Her skin is redundant at the base of the neck. Pelvic exam exhibits a small uterus and a solid left ovarian mass. She has no withdrawal bleeding after a progesterone challenge. Which of the following laboratory test findings would you expect in this patient? SerumFSH SerumLH —Serumestradiol Chromosomes A High Normal Low 45 B. — Nonmal High Low 46 Cc. High High Low 4s D. Normal Normal Normal 46 E Low Low Low 45 FSH = follicle simulating hormone, LH = luteinizing hormone 48 Note: This material is copyrighted. All rights reserved. (Edward F. Goljan, M.D.) 2001 Endocrine questions Items 1-3 Te RTUTBR | FTeindex | TSH aT | [AL [inereased_| Tnoreased | Increased | Decreased | increased | [Bo [increased [Decreased _| Normal. ‘Normal Not indicated | Teo Tinereased [Increased | Increased | Decreased | Decreased [D.[Decreased [Increased _| Normal ‘Normal. ‘Not indicated E- [Decreased | Decreased | Decreased _| Increased | Not indicated | RijU= resin T; uptake, T.BR = T binding ratio, "l= radioactive iodine uptake 1. A.45-year-old obese woman presents with a history of weight loss, weakmess, and palpitations that keep her awake at night. She states that she has lost over 50 pounds the last 8 months while attending a weight loss clinic. Physical exam reveals lid stare, a non-palpable thyroid gland, brisk deep tendon reflexes, systolic hypertension, and an iregularly iregular pulse. She is not taking any prescription medications other than the packet of pills given to her by the clinic. 2, A 22-year-old woman complains of intermittent fluttering in her chest. Physical exam reveals @ normal thyroid, no lid stare or exophthalmos, a regular heart rate of 108 beats/minute, normal deep tendon reflexes, and blood pressure of 100/80 mm Hg, A mid-systolic click and murmur is heard at the apex that increases with expiration. She is currently taking birth control pills. 3, A 28-year-old woman presents with complaints of chronic constipation and progressive weight gain over the last 6 months in spite of being on a pure vegan diet. She is currently on no prescription or Gver-the-counter medications. Physical exam exhibits a pale young woman with periorbital puffiness, dry, yellow-colored skin, normal sclera, a normal cardiovascular and respiratory exam, Selayed deep tendon reflexes, and proximal muscle weakness in her lower extremities. Items 4-5 Seram Caleium Note: the square represents normal values 4, A.55-year-old woman with diastolic hypertension presents with a sudden onset of right flank pain ‘vith radiation of pain into the right groin. A urinalysis reveals a positive dipstick for blood and a negative dipstick for nitrite and leukocyte esterase. Sediment exam exhibits numerous RBCs and square crystals resembling the back of an envelope. 5. Anewbom presents with tetany, heart failure, and an absent thymic shadow. 49 10. Which of the following clinical and laboratory abnormalities occur in BOTH Addison's disease and panhypopituitarism? ‘A. _ Low 11-deoxycortisol post-metyrapone test B. Normal urine for 17-ketosteroids C. Hypernatremia D. Hyperkalemia E. LowTSH Which of the following alterations in lipid metabolism is expected in a patient with diabetic ketoacidosis? ‘A. Activation of capillary lipoprotein lipase B, Increased B-oxidation of fatty acids C. Decreased production of acetyl-CoA D. _ Increased fatty acid synthesis, E. Inhibition of hormone sensitive lipase ‘A 2i-year-old woman has been amenortheic for the last 6 months. She complains of @ milky discharge from her nipples that has been present for the last 7 months. The pregnancy test is negative and the serum TSH is normal. Which ofthe following additional tests should you order on this patient? A. Serum prolactin B. Metyrapone test C. Serum Ty D. Serum cortisol E. Serum gonadotropins Which of the following tests distinguishes pituitary Cushings from both adrenal and ectopic Cushings? A. 2é-hour urine for 17-ketosteroids B. Low dose dexamethasone test C. Serum cortisol level D. High dose dexamethasone suppression test E, 24-hour urine for free cortisol ‘A 35-year-old pharmacist presents to your office with recurrent episodes of forgetfulness and firedness. A serum glucose is reported to be 20 mg/dL (70-110 mg/dL). Additional studies on the same sample reveal a high serum insulin and high C-pepide level. Based on these findings, you suspect the patient has... ‘a benign tumor involving B-islet cells an early phase of type I diabetes mellitus a benign tumor involving o-islet cells ‘ectopic secretion of an insulin-like factor surreptitiously injected human insulin mPOOwP 50 Note: This material is copyrighted. All rights reserved. (Edward F. Goljan, M.D.) 2001 ML. 'A.39 year old type I insulin dependent diabetic has a burning sensation around his ankles and on the bottoms of both feet. Neurological examination reveals depressed Achilles and knee jerk reflexes bilaterally and decreased light touch sensation in both lower extremities. The mechanism for this is most closely associated with... ‘A. osmotic damage B. thiamine deficiency C. pernicious anemia D. syringomyelia E, lumbar disk disease 12, Which of the following water deprivation test results would you expect in a patient with polyuria after head trauma? f POsm post water | _UOsm post ‘UOsm post deprivation _| water deprivation | _vasopressin_| normal 292, 750 760) [A 312 98 120 B 319 110 550, {c 288 760 730) 13, Which of the following most clearly differentiates type I from type Il diabetes mellitus? A. Insulitis B. Positive family history C. Hypoglycemic reactions D. _Post-insulin receptor defects E. Increased glycosylated hemoglobin 14. An 8-year-old boy presents with headache and bilateral hemianopsia. A CT scan of the skull reveals 15, enlargement of the sella turcica secondary to a cystic mass with multiple calcifications, Which of the following is an expected laboratory finding in this patient? Low insulin growth factor-I levels Hyponatremia and hyperkalemia ‘Normal urine for 17-hydroxycorticoids High serum TSH and low serum T, Normal serum 11-deoxycortisol after metyrapone moOpD ‘A 48-year-old man with diastolic hypertension complains of headaches, drenching night sweats, and palpitations that keep him awake at night. He has a family history of hypertension. Physical exam findings include an anxious man with a mean blood pressure of 152/96 mm Hg and a regular pulse of 148 beats/minute. There is no evidence of thyromegaly. Deep tendon reflexes are brisk. Which of the following tests is the best screening test for this patient? ‘A. Serum thyroid stimulating hormone B. 24-hr urine for metanephrines C. 24-hr urine for free cortisol D. Plasma aldosterone E. Metyrapone test sl Note: This material is copyrighted. All rights reserved. (Edward F. Goljan, M.D.) 2001 16. A2 year old child with newly discovered diastolic hypertension has multiple nodular lesions on the skin. A biopsy of one of the lesions exhibits an infiltrate of small, round, hyperchromatic cells. The cells stain positive for S100 antigen. A radionuclide bone scan is reported to have multiple lucencies consistent with metastatic disease. The primary origin of the patients skin and marrow infiltrate is from which of the following sites? Skin Bone Kidney Thyroid Adrenal cortex ‘Adrenal medulla mMoOwD 17. Which of the following biochemical reactions is most responsible for hyperglycemia in a patient with diabetic ketoacidosis? A. Glycogen P glucose B. Glycerol 3-phosphate > DHAP. C. Lactate ————p> pyruvate D. Alanine ————} pyruvate ~~ E. Citrate —————} Oxaloacetate —————>Malate «v0 DHAP = dityéroxyacetone phosphate 18. A. 26 yr old woman with an abruptio placenta develops hypovolemic shock requiring multiple units of packed red blood cells. Her baby is delivered without consequence. Two days later, the mother experiences a sudden cessation of lactation. The pathologic process most likely responsible for this complication is... hemorrhage into a preexisting pituitary adenoma increased secretion of dopamine coagulation necrosis of the anterior pituitary benign pituitary adenoma with secretion of prolactin hypothalamic dysfunction leading to hypopituitarism roOOw> 19. A 22 yrold type I diabetic develops sweating, anxiety, excessive hunger, and light-headedness. The frst tp inthe management ofthis patient iso. order a plasma Hebaic order a serum insulin and C-peptide level order serum electrolytes order a 24 hr urine for metanephrines give the patient a candy bar mOOWD> 20. 425 yr old man with non-tender thyromegaly and palpable cervical lymph nodes has an elevated serum calcitonin level. A fine needle aspiration of the thyroid is scheduled. He has a family history of thyroid disease, hypertension, and renal stones. Which of the following additional laboratory tests should be ordered on this patient? Serum electrolytes and 24 hr urine for free cortisol 24 br urine for metanephrine and serum PTH/calcium Serum aldosterone and 24 hr urine for potassium Serum gastrin and gastric analysis to measure basal acid output ‘Serum vasointestinal peptide and somatostatin levels moowp 52 pa AO Oc a Note: This material is copyrighted. All rights reserved, (Edward F. Goljan, M.D.) 2001 21. A2lyrold man with a history of infertility is noted to have a large varicocele in the left scrotal sac. His physician suspects a problem with the seminiferous tubules. Which of the following sets of laboratory test results would you expect in this patient? Serum Serum Serum FSH LH testosterone Sperm count A. High Normal Normal Decreased B. Normal High Low Decreased C. High High Low Decreased D. Normal Normal Normal Normal 22. A 45-year-old man with diastolic hypertension presents with muscle weakness and tetany. Laboratory studies reveal mild hypernatremia, hypokalemia, and metabolic alkalosis. There is no evidence of pitting edema or volume depletion. The patient most likely has... ‘Addison's disease chronic renal failure primary aldosteronism been taking a loop diuretic type I renal tubular acidosis, mOOM> Musculoskeletal and autoimmune questions 1. A sexually active 30 year old man presents with fever and pain in his right big toe that woke him up at night. He has been taking aspirin to relieve the pain without relief. His mother has severe osteoarthritis. The right toe is swollen, hot and exquisitely sensitive to touch. Laboratory studies reveal a neutrophilic leukocytosis and left shift. A synovial tap was performed. Based on the above history, his problem is most likely related to... ‘A. joint inflammation secondary to a positively birefringent crystal B. disseminated gonococcemia with septic arthritis C. _underexcretion of uric acid in the urine D. osteomyelitis secondary to hematogenous spread of Staphylococeus aureus E. an HLAB27 positive spondyloarthropathy 2, Which of the following characterizes joint disease associated with rheumatoid arthritis rather than osteoarthritis? ‘A. Cartilage fibrillation B. Subchonéral bone cysts C. Osteophytes D. _ Ankylosis of the joint E. _Bouchard’s nodes 3, A28 year old man who works in a summer camp in upstate New York presents with bilateral facial weakness involving both the upper and lower facial muscles. He also complains of joint pains in oth knees. He has a history a few weeks ago of having had a peculiar rash on his right thigh that was circular, red colored and had a central area of clearing. The patient's disease is most likely caused by a... ‘A. gram negative diplococcus B. _ spirochete C. gram positive coccus D. virus E. immune complex reaction 33 "Note: This material is copyrighted. All rights reserved. (Edward F, Goljan, M.D.) 2001 ‘A febrile, sexually active 23 year old woman presents with a hot, swollen right knee and pustular lesions on the palm of her left hand. She recently retumed frome camping trip in Colorado. A gram Stain ofthe synovial fluid in the knee and from an aspirate of one of te pustules revealed a... gram positive diplococcus B. gram negative rod C. spirochete D. gram negative diplococcus E. intranuclear inclusion p> A. Yersinia pestis B. Eikenella corrodens C. Fusobacterium D. Pasteurella multocida E. Staphylococcus aureus ‘Which ofthe following clinicopathologc findings is commonly present in both theumatoid arthritis and ankylosing spondylitis? A. Sacroiliitis B. Male dominance Cc. Ankylosis of the joint D. Overgrowth of synovial tissue E. Autoantibodies against immunoglobulins for back pain. You would expect an x-ray of the patients back to revenl A. radiodense lesions in the vertebra B. _ osteophytes at the joint margins C. vertebral compression fractures D. lytic lesions in the vertebra E. fusion of the vertebra his current rheumatologic problems? A. Overproduction of uric acid B. Invasive gastroenteritis C. Colorectal cancer D. Ulcerative colitis E. Crohn's disease 54 Note: This material is copyrighted. All rights reserved. (Edward F. Goljan, M.D.) 2001 9. 3 year old girl has an acute onset of spiking fever, rash, generalized painful lymphadenopathy, hepatosplenomegaly, and polyarthritis. The cardiac exam is normal, A serum antinuclear antibody test, rheumatoid factor, and urinalysis are reported as negative. A CBC reveals an absolute neutrophilic leukocytosis, mild normocytic anemia, and a normal platelet count. Which of the following best characterizes the nature of this patient's disease? ‘A. Non-disabling, non-infectious inflammatory joint disease B. _ Progressively disabling infectious joint disease due to a bacteria C. _ Progressively disabling non-infectious inflammatory joint disease D. _ Non-disabling infectious joint disease due to an immunologic reaction against a bacteria E. _Progressively disabling non-infectious inflammatory joint disease associated with an HLA- Bz haplotype 10, Which of the following clinicopathologic findings occurs in both osteoarthritis and rheumatoid arthritis? C. Narrow joint space D. _Subchondral bone cysts E. Metacarpophalangeal joint involvement 11. 45 year old man has chronic myelogenous leukemia that has progressed into an acute blast crisis. He is currently being treated with multiple antileukemic agents. One week into therapy, he develops oliguric renal failure. A urinalysis reveals an acid pH and numerous crystals. Which of the following best explains the mechanism for this patient's renal failure? Drug nephrotoxicity Overproduction of uric acid Leukemic infiltration of the kidneys Underactivity of 5-phosphoribosyl-I-pyrophosphate (PRPP) Overactivity of hypoxanthine-guanine phosphoribosyltransferase (HGPRT) POOD> 12, A 65-year-old man enters the examining room with broad-based ataxia. Positive neurological findings include positive Romberg test, loss of pain and vibratory sensation in the lower extremities, and absent deep tendon reflexes in the lower extremities. Both pupils accommodate but do not constrict with direct light stimulation. The patient's right knee is swollen and soft tissue swelling is noted around the joint. An x-ray reveals extensive resorption of bone. Which of the following best explains the pathogenesis of this patient's joint disease? A. Septic arthritis B. _ Loss of pain sensation C. _ Immunocomplex disease D. Upper motor neuron disease E, _ Cavitary lesion in the cervical spinal cord 13. Disabling joint disease, nodular lesions in the hung associated with dust borne diseases, xerostomia, and splenomegaly characterize a theumatologic disease with which one of the following laboratory abnormalities? ‘A. Positive serum antinuclear antibody with a rim pattern B. Positive band test on a skin biopsy C. _Anti-ribomucleoprotein antibodies D. _Anti-centromere antibodies E, [gM antibodies against IgG 55 ‘Note: This material is copyrighted. All rights reserved. (Edward F. Goljan, M.D.) 2001 14, 15. 16. 1. |A.32year old pregnant woman, who is under treatment for hypertension with hydralazine, develops ‘an abrupt onset of a left-sided pleural effusion, photophobia, ‘and joint pains in both of her hands. Which of the following antibodies is most likely present in this patient? ‘A. Anti-double stranded DNA B. Anti-centromere CC. Antichistone D. — Anti-Scl-70 E, Anti-Sm |A.24 year old woman with a previous history of moming stiffness of both hands presen Hott dyspnea, neck vein distention with inspiration, and muffled heart sounds, Urinalysis reveals RBC cae hematuria, and mild to moderate proteinuria. A serum antinuclear antibody test is positive and has a titer of 1:1280. Based on these findings, the patient most likely has... ‘A. mixed connective tissue disorder B. _ progressive systemic sclerosis C.__ systemic lupus erythematosus D. juvenile rheumatoid arthritis E. adult rheumatoid arthritis \Which of the following clinicopathologic findings is present in both systemic lupus erythematosus and Sjogren's syndrome? A. Sclerodactyly B. Positive syphilis serology CC. Anti-SS-A (Ro) antibodies D. _Norrinfectious endocarditis E. _ Dysphagia for solids and liquids Which of the following relationships are correctly matched? SELECT 5 Noter ths is not style of question on the USMLE, however itis an excellent learning tool for commonly asked small topics ‘A. Osteomyelitis ‘Most common cause in sickle cell disease is Staphylococcus aureus B. Osteomyelitis Most common site is the metaphysis. Most common mechanism for spread to bone in children is local extension from a soft tissue abscess. C. Chronic osteomyelitis Orifice of sinus tract is subject to developing squamous cell carcinoma D. _ Potts disease ‘Staphylococcus aureus involving the vertebral colurmn F._Lyme's disease Most common late manifestation is arthritis. Bilateral Bell's palsy may occur. G. __Osteopetrosis AD disorder with an osteoblastic defect. Too little bone. Pathologic fractures, deafness. HL Osteogenesis imperfecta AD disorder with a defect in synthesis of type J collagen. Blue sclera due to underlying choroidal veins. Pathologic fractures. L Osteoporosis ‘Most common cause in women is estrogen lack. More bone is lost by osteoclastic activity than replaced by osteoblastic activity. J Osteoporosis Most common fractures are vertebral compression fractures Osteoporosis Routine bone x-rays are the most accurate method for diagnosing the condition 56 Note: This material is copyrighted. All rights reserved. (Edward F. Goljan, M.D.) 2001 18. 19. ‘Which of the following relationships are correctly matched? SELECT 5 A. Benefits of estrogen osteoporosis Femoral neck fracture Scaphoid bone fracture Colles’ fracture mop p ‘Supracondylar fracture F. Aseptic necrosis Legg-Perthes ZO in Maintains the high HDL and low LDL. Increases bone density rather than preventing loss of bone density. Danger of endometrial cancer if progesterone is not added. Danger of aseptic necrosis from compromise of medial femoral cireumflex artery by a posterior dislocation Danger of osteomyelitis. Pain in the anatomical snuff box. Dinner fork deformity of the distal radius. Common fracture when falling on the outstretched hand. Fracture of the distal humerus. Median nerve may be injured. Brachial artery compromised leading to possible ischemic contracture of forearm muscles. Most common cause is sickle cell disease. Bone x-ray reveals increased density. MRI is the most sensitive test. Osteomyelitis of the femoral head in children < 10 yrs old Osgood-Schlatter's disease Inflammation of the proximal tibial apophysis with repair resulting in knobby knees. Leads to problems with bone growth in the leg. Which of the following relationships are correctly matched? SELECT 5 A. Paget's disease of bone Osteochondroma Ewing's sarcoma Chondrosarcoma Osteoma, ™ m 9 OD Osteogenic sarcoma 2 ‘Muscle weakness H. Dupuytren's contracture L Desmoid tumor ‘Bone thickens and fractures easily. Serum alkaline phosphatase is elevated. Complications include osteogenic sarcoma, pathologic fractures, high output cardiac failure. Most common overall benign bone tumor "Round (small) cell tumor" with x-ray findings of "onion skinning". Primarily affects young adults. ‘Most common malignant cartilaginous tumor. Grade determines it biologic behavior. Most commonly affects pelvic bones. Radiograph demonstrates a radiolucent defect surrounded by densely sclerotic bone. Nocturnal pain is relieved by aspirin ‘Association with Rb suppressor gene on chromosome 13. "Sunburst appearance" and "Codman's triangle" on x-ray. Primarily affects the pelvic bones. ‘Causes include upper/lower motor neuron disease, primary muscle disease, myasthenia gravis, and extrapyramidal disease Fibromatosis involving palmar tendon sheaths. Related to hyperestrinism in cirrhosis. Fibrosarcoma of the abdominal wall often associated with Turcot's syndrome 37 Note: This material is copyrighted. All rights reserved. (Edward F. Goljan, M.D.) 2001 20. Which of the following relationships are correctly matched? SELECT 2 ‘A. Embryonal rhabdomyosarcoma Most common adult striated muscle sarcoma. Presents 1s a vaginal mass in women or bladder/prostate mass in men. B. _Lipoma Most common benign soft tissue tumor in women C.— Leiomyoma Most common benign soft tissue tumor of the uterus and gastrointestinal tract D. _Rhabdomyomas ‘Most common benign tumor of the heart in children with tuberous sclerosis E Malignant fibrous histiocytoma Most common childhood soft tissue sarcoma. ‘Associated with radiation exposure and scars Skin questions 1. A 52-year-old man with congestive heart failure develops cough and swelling in the deep subcutaneous tissue. He is most likely taking. ‘A. athiazide diuretic B. an ACE inhibitor C. aphenothiazine D. digitalis E. — acalcium channel blocker 2. The most common fungal cause of tinea capitis with a negative Wood's lamp is... ‘A. Trichophyton tonsurans B. Microsporum canis C. Trichophyton rubrum D. Candida albicans - E. Malassezia furfur 3. _A43-year-old woman presents with joint pains and a buterfly-like rash on the face. She is most likely taking... A. thiazides B. oral contraceptives C. hydralazine D. doxycycline E. barbiturates 4, A 42-year-old man has recurrent development of vesicular and bullous lesions in sun-exposed areas. He has had to avoid alcohol, because it seems to coincide with these episodes. You would expect this patient to hav a history of abdominal pain an increase in 8~aminolevulinic acid in his urine an increase in porphobilinogen in his urine colorless urine during these attacks a decrease in red blood cell uroporphyrinogen decarboxylase MOODP 58 Note: This material is copyrighted. All rights reserved. (Edward F. Goljan, M.D.) 2001 5. A 30-year-old female with a long history of alcohol and barbiturate abuse presents with diffuse colicky abdominal pain. Examination reveals numerous surgical scars on the abdomen. There is no history of photosensitivity. You suspect that her urine would. ‘A. contain an increase in uroporphyrin B. contain an increase in coproporphyrin C. be positive for blood D. _ have an excess amount of urobilinogen E. _ tuma port wine color after exposure to light 6. A baseball player develops a vesicular rash around the neck and on his forearms. He is most likely taking which of the following drugs? Thiazide diuretic ACE inhibitor Tetracycline Penicillin Calcium channel blocker MOO D> 7. _A25 yrold sexually active man has a verrucoid appearing lesion on the shaft of his penis. The infectious agent that is responsible for this penile lesion belongs to which of the following subtypes of microbial pathogens? A. Spirochete B. Rickettsia C. Chlamydia D. Fungus E. Vins 8. _A35 yrold black man has scaly lesions on his scalp and raised, plaque-like lesions on his elbow and trunk, The lesions have silver-colored scales on the surface. Which of the following is a ‘common clinical finding associated with this skin lesion? A. Positive Nikolsky sign B. Nail pitting C. Dermatographism D. Wickham's stria E. Squamous cell carcinoma 9. Which of the following most determines the prognosis of 2 superficial spreading malignant melanoma? ‘A. Gender of the patient B. Depth of invasion C. Duration of exposure to sunlight D. Size and color variation E. Presence of a radial growth phase 39 Note: This material is copyrighted. All rights reserved. (Edward F. Goljan, M.D.) 2001 10. u 12, 1B. 14, An 4 month old infant has a weeping, eczematous rash on the cheeks. The child is a mouth breather and constantly has a stuffy nose and otitis media infections. Which of the following characterizes the pathogenesis of this lesion? A. Autoantibodies against DNA B. Type I hypersensitivity reaction C. Type Il hypersensitivity reaction D. Type IV hypersensitivity reaction E, Superficial dermatophyte infection A.49 yr old woman has a scaly, eczematous skin lesion related to the nickel in the metal clip on her girdle. The pathogenesis ofthe skin lesion is similar to the pathogenesis of. a wheal and flare reaction associated with a bee sting a maculopapular rash on a patient taking penicillin palpable purpura in a small vessel vasculitis the rash in poison ivy tinea corporis, PoOw> A.25 yr old woman is noted to have pruritic circular lesions on her leg. They have a red, scaly border and a central area of clearing. Which of the following should be your first step in the diagnostic work-up of this patient? A. Punch biopsy B. KOH preparation C. Tzanck preparation D. _ Peripheral blood count for eosinophils E. Urine sample for porphyrin analysis ‘A 4 yr old child has multiple crateriform lesions with "sandy-like" material in the area of umbilication located over the trunk. Which of the following best describes the pathogenesis of these lesions? A. Viral infection B. Ultraviolet light C. Fungal infection D. Bacterial infection E. Type hypersensitivity reaction A53 yr old farmer has raised, pearly gray lesions on the dorsum of his hands and forearms. Which of the following best describes the pathogenesis of the pearly white lesion on the hand? Viral infection Ultraviolet light Fungal infection Bacterial infection ‘Type I hypersensitivity reaction moowp i te Note: This material is copyrighted. All rights reserved. (Edward F. Goljan, M.D.) 2001 15, 2S yrold woman complains of multiple areas of skin that do not tan during the summer. Physical examination reveals areas of hyperpigmentation and hypopigmentation on her chest and back. A’ KOH preperation taken from a the border of a hypopigmented skin lesion reveals yeasts and hyphae. Which of the following skin disorder groups is associated with the pathogen noted in the KOH preparation? ‘Tinea capitis : Pityriasis rosea Sebortheic dermatitis : Tinea versicolor Tinea corporis : Onychomycosis Pityriasis rosea : Tinea versicolor Onychomycosis : Seborrheic dermatitis moowP 16. A 26 yr old man presents with slightly pruritic oval-shaped lesion on the trunk that has ccythematous borders and a pale center. A few days later, arash develops on the trunk that follows the lines of Langer. The patient most likely has... ‘A. a superficial dermatophytoses B. a viral exanthem CC. pityriasis rosea D. an urticarial reaction E. contact dermatitis 17, A febrile 8 yr old boy has a rash on the face, trunk, and extremities. Some of the lesions are flat, ‘Dhile others are vesicular, and others are pustular. A Tzanck prep is performed and reveals jnultinucleated squamous cells with intranuclear inclusions. The patient most likely has... A. an HSV-1 infection B. varicella Cc. variola D. Herpes zoster E. drug reaction 18, 72 yr old man with chronic lymphocytic leukemia has @ painful, vesicular rash located on the Shoulder. It begins at the midline of the neck and extends over the deltoid area. The pathogenesis of the skin lesion in this patient is attributable to... reactivation of a latent viral infection an allergic contact dermatitis an ultraviolet light-related dermatitis a group A streptococcus infection invasion of skin by leukemic cells POND 19, A 16yr old boy presents with yellow, crusted lesions over the face, neck, and upper body. Ten days ‘Ther the rash, the patient develops periorbital puffiness, hypertension, oliguris, and smoky colored Urine with hematuria and RBC casts. The pathogenesis of the hypertension and urinalysis finding in the patient is most closely associated with... sepsis due to Streptococcus pneumoniae sepsis due to Streptococcus pyogenes sepsis due to Staphylococcus aureus an immunocomplex disease associated with group A streptococcus ‘an immunocomplex disease associated with antibodies against DNA moaw> 61 Note: This material is copyrighted. All rights reserved. (Edward F. Goljan, M.D.) 2001 20. A.17 yr old male presents with facial lesions characterized by erythematous papular lesions with ‘white and black centers. Some of the lesions have a nodular-cystic pattern. The pathogenesis of the inflammatory component of the facial lesions in this patient is most closely associated with... ‘a group A streptococcal infection folliculitis secondary to Staphylococcus aureus an immunocomplex disease associated with antibodies against DNA production of lipases by Propionibacterium acnes folliculitis secondary to Streptococcus pyogenes moom> CNS and special senses questions 1. Cognitive abnormalities, ataxia, nystagmus, ophthalmoplegia, and foot drop are all associated with... A. folate deficiency B. multiple sclerosis C. Alzheimer’s disease D. alcoholism E. Parkinson's disease 2. A family history of chronic liver disease beginning at an early age and a movement disorder developing later in life characterizes a disease associated with. a triplet repeat mutation low ceruloplasmin levels thiamine deficiency excess alcohol intake vitamin B,2 deficiency 3. A 40-year-old neuropathologist developed a rapidly progressive dementia and died. The Bathogenesisofhis disorder i related to... a decrease in acetylcholine levels a deficiency of dopamine neuronal damage by amyloid a slow virus disease involving prions subacute sclerosing panencephalitis mooo> POOwP 4, A 15-year-old boy develops fever, nuchal rigidity, and petechial lesions. A spinal fluid reveals increased protein, decreased glucose, increased neutrophils, and a positive gram stain. You would expect the CSF gram stain to reveal... A. gram positive diplococei B. _ gramnegative coccobacilli C. gram positive rods D. gram negative diplococei E.__ gram positive cocci 5. Which of the following represents a primary brain tumor that would more likely develop in a child rather than an adult? A. Glioblastoma multiforme B. Medulloblastoma C. Malignant lymphoma D. Acoustic neuroma E. Meningioma 62 Note: This materi! 1s copyrigotea, Ail nights reserved. Usuward b. Goytiy hay 2 6. An afebrile 52 year old smoker with weight loss, cough, and hemoptysis, develops a severe headache associated with unilateral lid lag, ophthalmoplegia, and mydriasis. An MRI reveals multiple densities in the cerebral cortex. The pathogenesis of this patient's neurologic condition most closely relates to. 1 demyelinating disease primary CNS tumor embolic stroke ‘cerebral edema with uncal hemiation cerebellar herniation into the foramen magnum roop> 7. An autopsy is performed on a 40-year-old man with dementia. Examination of the brain reveals atrophy of the frontal and temporoparietal lobes and senile plaques on histologic examination of the tissue. The pathogenesis of the CNS disease in this patient is most closely related to.. A. B-amyloid protein B. lacunar infarcts C. decreased dopamine levels D. aslow virus disease E. _ a triplet repeat disorder 8. Recurrent episodes of blurry vision, scanning speech, and paresthesias is most closely associated with. A. multiple sclerosis B. Wemicke's encephalopathy C. slow virus disease D. Huntington's disease E. — Wilson's disease 9. Anencapsulated mass is removed from the right cerebellopontine angle of a 32-year-old male with right-sided sensorineural hearing loss and facial numbness in the distribution of the trigeminal nerve. The patient most likely has a/an... A. ependymoma B. medulloblastoma C. demyelinating disease D. meningioma E, acoustic neuroma 10. A 45-year-old woman with a history of mitral stenosis and a chronic arrhythmia died suddenly at home. At autopsy, a hemorthagie lesion was noted at the periphery of the temporal lobe. The patient most likely has a/an.. A. atherosclerotic stroke B. intracerebral hematoma C. embolic stroke D. glioblastoma multiforme E. AV malformation 63 Note: This material is copyrighted. All rights reserved. (Edward F. Goljan, M.D.) 2001 11. A 62-year-old man complains of weakness in his left arm and tremor in his hands that prevents him from writing legibly. He has a shuffling gait when he enters the examining room. His deep tendon Teflexes are normal. The patient's clinical findings most closely relate to.. A. amyotrophic lateral sclerosis B. atherosclerotic stroke C. pure motor stroke D. Parkinson's disease E, _ post-poliomyelitis syndrome 12, A 30-year-old woman states that she burns her hands without feeling any pain. Physical exam reveals decreased pain and temperature sensation in the upper extremities, atrophy of the intrinsic muscles of his hand, and abnormal deep tendon reflexes in the upper extremity. The patient most likely has... A. syringomyelia B. multiple sclerosis C. amyotrophic lateral sclerosis, D. _ spinal cord tumor E. By deficiency 13. A febrile 28-year-old man with AIDS and a CD, T helper count of 50 cells/uL develops focal epileptic seizures. A CT scan reveals multiple ring enhancing lesions in the brain. The pathogenesis of this patient's CNS disorder is most closely related to. A. cytomegalovirus B. cryptococcosis C. toxoplasmosis D. Epstein-Barr virus E, disseminated MAI 14, Which of the following is more often associated with Escherichia coli than Pseudomonas aeruginosa? A. Osteomyelitis from puncture wounds through rubber footwear Malignant otitis externa in a diabetic patient Pneumonia in an intensive care unit Death in a patient in a burn unit Meningitis in a newborn 15. _A.55 year old man has broad-based ataxia and loss of pain and temperature sensation in the lower extremities. Both pupils accommodate but do not constrict with direct light stimulation. You would expect the spinal fluid to exhibit... ‘A. encapsulated yeast with narrow-based buds B. —aneutrophil dominant cell count C. apositive VDRL D. spirochetes E. _ xanthochromia Note: This material is copyrighted. All rights reserved. (Edward F. Goljan, M.D.) 2001 16. 11, 18. 19. 20, ‘A biopsy finding in the cerebellum from @ comatose 28 yr old man revealed Purkinje cells with an osinophilie inclusion. The patient worked in a wild animal park. The vector most likely responsible for this patient's disease isa... A. dog B. bat C. raccoon D. — mosquito E. skunk patient as well as other members of his family has skin Sndings characterard Dy ox febtling and multiple pedunculated, pigmented polypoid lesions. In a¢diton, there are fawn colored oval lesions noted as well. A few of the family ‘members have hypertension and sensorineural hearing loss. This patient most likely has... ‘A. tuberous sclerosis B. the dysplastic nevus syndrome Cc. acanthosis mgricans D. multiple seborrheic keratoses E.__ neurofibromatosis A spinal tap in a 28 yr old man with AIDS has numerous’ yeast forms with a narrow based bud. You ani expect which ofthe following groups of spinal fluid results i this pation”? Note: the square represents the normal values for CSF glucose and CSF protein B e CSF protein » Cb CSF glucose ‘An autopsy performed on a 29 yr old man reveals a necrotic frontal abe abscess. Histologic exam A ais numerous wide-angled, non-septate byphae. Based on the morphology of the pathogen, the patient most likely was... an alcoholic acave explorer a missionary in Africa ‘an insulin dependent diabetic ‘a pig farmer who ate raw bacon mpOB> [AZ yrold man, who died in a car accident, had a large clot in the epidural space overlying the left hemisphere, Which of the following is the most common mechanism responsible for the autopsy finding in this patient? Cerebral ischemia with neuronal degeneration Rupture of @ congenital aneurysm Skull fracture with an arterial bleed Skull fracture with a venous bleed Embolism ftom the left heart ROOD Note: This material is copyrighted. All rights reserved. (Edward F. Goljan, M.D.) 2001 2. 23. 24. 25. ‘An elderly patient in a nursery home fell and hit her head. She died 3 days later in the hospital. At autopsy, large blood clot overlying the right hemisphere was noted between the dura and arachnoid membranes. Which of the following best explains the autopsy finding in this patient? Cerebral ischemia with neuronal degeneration Rupture of a congenital aneurysm Skull fracture with an arterial bleed ‘Skull fracture with a venous bleed Embolism from the left heart MOOD A 58 yr old man with severe diabetes mellitus died from a stroke. At autopsy, a large blood clot was noted in the area of the putamen and globus pallidus. Which of the following best explains the mechanism for the autopsy finding in this patient? A. Cerebral ischemia with neuronal degeneration B. Rupture of a congenital aneurysm C. Arteriovenous malformation D. _ Embolism from the left heart E. _ Poorly controlled hypertension A.58 yr old man with adult polycystic kidney disease experienced a severe occipital headache and died. At autopsy, the surface of the brain was covered by blood. Which of the following is the most often responsible for initiating the autopsy finding in this patient? A. — Rupture ofa congenital aneurysm B. _ Internal carotid artery atherosclerosis c. Arteriovenous malformation D. _ Embolism from the left heart E. _ Neoplastic process involving astrocytes An autopsy finding from a 35 yr old man with severe frontal headaches revealed a hemorrhagic and necrotic mass that traversed the corpus callosum and involved both sides of the brain. Which of the following best describes the pathogenesis of this patient's brain lesion? Benign neoplasm derived from oligodendrocytes Malignant neoplasm derived from astrocytes Intracerebral hematoma due to hypertension. Embolism from the left heart Atherosclerotic stroke PoOw> ‘An incidental finding noted at an autopsy performed on a 45 yr old woman was a hard, popcom shaped lesion attached to the dura. It impinged on the cerebral cortex but did not invade brain tissue. Which of the following best describes the derivation of this brain lesion? A. Astrocytes Oligodendrocytes Ependymal cells Microglial cells Arachnoid granulations moow 66 PAARARARAARARAAAEHARARAOE RARER AAA Note: This material is copyrighted. All rights reserved. (Edward F. Goljan, M.D.) 2001 26. Unilateral papilledema would most likely be associated with which one of the following clinical conditions? A. Glaucoma B. Uncal herniation C. Multiple sclerosis D. Alzheimer's disease E. Hypematremia 27. Which of the following clinical scenarios correlates best with the rapid increase in primary central nervous system malignant lymphomas in the United States? Increase in Epstein-Barr virus infections Increase in human immunodeficiency virus infections Increase in prion-related central nervous system disease Increase in slow virus diseases in immunodeficient patients Increase in radiation therapy of metastatic cancers to the brain moOw> 28. One week after an upper respiratory infection, a 25-year-old man develops weakness in the lower legs that is progressing into his upper torso. A spinal tap report indicates increased protein, a normal glucose, a negative gram stain, and 20-30 lymphocytes/mononuclear cells/}iL. Which of the following categories of disease best explains the clinical and laboratory findings in this patient? A. Inborn error of metabolism B. Atherosclerotic disease CC. Demyelinating disease D. Neoplastic disease E. Infectious disease 29. A 32-year-old woman complains of recurrent episodes of ringing in the right ear along with a sense of fuliness behind the ear drum. She occasionally experiences bouts of dizziness where "the room seems to spin around". On physical exam, the Weber test lateralizes to the left ear and the Rinne test demonstrates that air conduction is longer than bone conduction in both ears. The tympanic ‘membrane has @ normal light reflex. Horizontal nystagmus is present in the right eye. Which of the following best explains the pathogenesis of this patient's disorder? Right middle ear infection ‘Tumor of the acoustic nerve Increased endolymph in the inner ear Brain tumor in the cerebellopontine angle Demyelinating disease involving the acoustic nerve BOOB> 30. A few weeks ago, a 65 year old man presented with a painless unilateral loss of vision described as 2 “curtain suddenly going down and then coming up.” He now presents with a sudden onset of ‘expressive aphasia, and contralateral hemiparesis and sensory loss. CT scans of the brain on admission and after 24 hours reveal no evidence of hemorthage. Which of the following pathologic processes best explains the pathogenesis of this patient's clinical findings in the past and present? Embolism Atherosclerosis ‘Lacunar infarction Intracerebral bleed Subarachnoid hemorthage moOw> 67 Note: This material is copyrighted. All rights reserved. (Edward F. Goljan, M.D.) 2001 31. 32. 33. 34, A 45 yr old wife of a pig farmer develops focal new onset epileptic seizures. Her husband has a habit of eating raw bacon. An MRI of her brain reveals multiple calcified cysts. Which of the following applies to the pig, the husband, and the patient regarding the pathogen responsible for her disease? Pig Husband Wife (patient) Definitive host Intermediate host Definitive host Intermediate host Definitive host Intermediate host Definitive host Intermediate host Intermediate host Intermediate host Definitive host Definitive host Intermediate host Intermediate host. Intermediate host ppoE> A normotensive 29-year-old man with AIDS is beginning to have significant visual loss in both eyes. His CD, T helper count is 50 cells/uL. Retinal exam exhibits cotton wool exudates in both eyes. The infectious agent that is responsible for this patient's central nervous system disorder belongs to which of the following subtypes of microbial pathogens? A Virus B. Fungus C. Bacteria D. Helminth E. — Sporozoan A 28-year-old man, who lives in the Northeast, Presents with a sudden onset of drooping and drooling out of both sides of his mouth, inability to close both eyes, and slurred speech. His wife states that a few weeks ago he removed a tick from his Tight thigh. In a few days following removal of the tick, a concentric, erythematous rash developed in that same area. The infectious agent that is responsible for this patient's previous skin disorder and current neurological findings belongs to which of the following subtypes of microbial pathogens? A. Virus B. Fungus C. Rickettsia D. Protozoan E. — Spirochete A 2-month-old infant has a progressive increase in head circumference. The pediatrician suspects hydrocephalus and orders an MRI. The MRI reveals dilatation of the third ventricle and lateral ventricles. Which of the following is the most likely cause of the patient's condition? Obstruction of the arachnoid granulations Obstruction of the aqueduct of Sylvius Dandy-Walker syndrome Amold-Chiari syndrome Tuberous sclerosis moOm> 68 Note: This material is copyrighted. All rights reserved. (Edward F. Goljan, M.D.) 2001 35, A.45 yr old man notices weakening in his ability to open jars with his right hand. He also notes “twitching” in the muscles of his legs and back and an inequality in the size of his calf and thigh muscles, with the muscles on the left much smaller than ‘those on the right. Physical exam reveals {trophy of the intrinsic muscles of both hands, the right greater than the left and absent deep tendon reflexes in the upper extremity and left leg. Muscle fasciculations are noted in the right forearm, left talf, and left thigh. The Babinski sign is negative, Sensory function is intact. Which of the following pathologic processes best explains the pathogenesis of the patient's neurological findings? Triplet repeat disorder Demyelinating disease Degeneration of anterior horn cells Metabolic disorder involving copper ‘Degenerative changes in the cervical spinal cord mOOBP ANSWERS AND DISCUSSIONS TO PATHOLOGY QUESTIONS Optional General principles in laboratory medicine questions 1 Answer: B- mean of the test is 20 mg/dL, 2 SD = 10 mg/dl, therefore 1 SD = 5 mg/dL 2. Answer: D- increasing sensitivity (less FNs) by lowering the reference interval automatically increases the predictive value of a negative test result ‘Answer: C: group A = all TNs, group B = TNs + FNs, group C= FPs + TPs, group D= all TPs 4. Amswer: A- estrogen increases the synthesis of thyroid binding globulin. This automatically increases the total T, level, since TBG has T4 bound to it. However, the free T4 level remains ‘unchanged. A similar question on the USMLE was involved an increase in serum cortisol in @ pregnant woman who had no signs of Cushing's syndrome. Estrogen also increases the synthesis of Transcortin, the binding protein for cortisol, hence the increase in serum cortisol, but no signs of hypercortisolism since free cortisol levels are normal. Cell Injury questions 1. Answer: E~ endocrine stimulation of target tissues is invariably hyperplasia. Choice A is atrophy, choice B is hypertrophy, choice C is metaplasia, choice D is hypertrophy 2, Answer: E~ hemorrhagic infarction. Choices A, B, and C are liquefactive necrosis, Choice D is atrophy, ‘Answer: D~ choice A has a low SaO;, choices B, C, and E have a normal PO: and S202, choice 4. Answer: B- goblet cells in the mainstem bronchus are an example of hyperplasia. If they were in the terminal bronchiole, it would be metaplasia. Choices C and D are hyperplasia. Choice E represents granulomatous inflammation. Inflammation questions 1. Answer: D~ patient with MPO deficiency would have a respiratory burst but would still have @ microbicidal problem. In addition, there are no azurophilic granules in the cytoplasm of neutrophils, 2. Answer: B- inhibition of PLA: leads to 2 reduction in synthesis of both prostaglandins and leukotrienes. = this is a pericardial friction rub 4. Answer: D- note, C. difficile pseudomembranous colitis has a lot of similarities with diphtheria i) Note: This material is copyrighted. All rights reserved. (Edward F. Goljan, M.D.) 2001 Answer: A~ cellulitis is usually due to group A strep, since hyaluronidase (spreading factor) is produced and allows the infection to spread through the subcutaneous tissue. Staphylococcus aureus produces coagulase, which changes fibrinogen into fibrin and localizes the infection leading to an abscess with drainage to the surface through a sinus. ‘Answer: D- without adhesion molecules, neutrophils cannot adhere to the endothelial cells in the venules, hence they cannot emigrate into tissue. For the umbilical cord to fall off, it must have neutrophil infiltration of the tissue leading to necrosis. Answer: B ‘Answer: D- causes them to release preformed chemicals like histamine and serotonin ‘Answer: C- recall that eosinophils have crystalline material in their red granules Fluids and hemodynamics questions including optional acid-base questions Answer: A- due to the decrease in TPR from vasodilatation (histamine, bradykinin, nitric oxide, release of anaphylatoxins) ‘Answer: B~ in cirrhosis, there are 2 alterations in Starling's forces~ an increase in hydrostatic pressure from portal vein hypertension and a decrease in synthesis of albumin, which decreases ‘oncotic pressure. Choices A and D are examples of exudates. Choice C is lymphedema. Choice E is an increase in hydrostatic pressure causing a transudate ‘Answer: D~ patients with heart failure and pitting edema have an excess of salt and water, hence these must be restricted. ‘Answer: D- the patient has inappropriate ADH syndrome. Since ADH reabsorbs solute free water from the kidneys, the excess water distributes in both the ECF and ICF compartments. Dilutional hyponatremia favors an osmotic gradient moving water into the ICF compartment. The Rx of choice is to restrict water, since the TBNa is normal. ‘Answer: C- the patient is in hypovolemic shock, hence the cardiac output is decreased, SVR is increased due to catecholamines, vasopressin, and AT Il, the MVO2 is decreased owing to the reduction in blood flow through the tissue, and the PCWP is decreased, since plasma volume is decreased. MVO; PCWP svR Cardiac output Decreased Decreased Increased _ Decreased MVO; PCWP svR Cardiac output Interpretation A. Normal Normal Normal Normal Normal B. Decreased Increased Increased Decreased Left heart failure D. Increased Decreased Decreased Decreased Septic shock ‘Answer: A- the patient has traveler's diarrhea, which is a secretory diarrhea with a loss of isotonic fluid. Serum sodium is normal, hence POsm is normal and there is no osmotic gradient. Answer: D- the patient has both left (bibasilar rales) and right heart failure (dependent pitting edema). In heart failure, there is an increase in TBNa and TBW, the TBNa is restricted to the ECF ‘compartment leading to pitting edema (ECF compartment is expanded due to the gain in salt), while the excess in TBW is distributed between the ECF and ICF compartments by osmosis. Since there is a greater increase in TBW than TBNa, patients have hyponatremia causing water to move into the ICF compartment (expanded), ‘Answer: E- the patient has central diabetes insipidus due to severance of the pituitary stalk from hhis motorcycle accident. There is a loss of pure water, therefore the patient develops hypematremia (increased POsm), which stimulates thirst. A pure water loss does not produce any signs of volume depletion, since the TBNa is normal. Since there is a loss of fluid, the ECF compartment is contracted and the osmotic gradient favors movement of water out of the ICF (contracted) into the ECF compartment. 70 ‘Note: This material is copyrighted. All rights reserved. (Edward F. Goljan, M.D.) 2001 12, 13, 14, 15. 16. 7. 18. 19. 20. 21 23 24. ‘Answer: F— the patient has a hypertonic gain of salt from the excessive amounts of sodium bicarbonate received during a cardiac resuscitation. He has signs of both left and right heart failure. ‘A hypertonic gain results in hypernatremia and an increase in POsm. The gain in TBNa is primarily in the ECF compartment, so it is expanded. Hypematremia favors an osmotic gradient with the ‘movement of water from the ICF into the ECF, hence the ICF compartment is contracted. ‘Answer: C- the patient has an isotonic gain of fluid, hence the serum sodium and POsm remain normal. There is no osmotic gradient, so the fluid expands the ECF compartment. Answer: B- the patient has a hypertonic loss of fluid, hence the presence of hyponatremia and a decrease in POsm. Since the hypertonic fluid is lost in the urine, the ECF compartment is contracted. Hyponatremia favors an osmotic gradient of water moving into the ICF, so it is expanded. Answer: E~ sweating results in a hypotonic loss of more water than salt, hence hypernatremia and increase in POsm is expected. Since there is a loss of hypotonic fluid, the ECF compartment is contracted. Hypernatremia favors the movement of water out of the ICF into the ECF compartment, hence it is contracted. Note the difference between the patient in question 8 with diabetes insipidus vs this patient. Due to loss of salt in this patient, there are signs of volume depletion, while the patient with CDI is not volume depleted, since salt is not lost. Answer: A- in DKA, the excess glucose increases the POsm leading to a gradient favoring the movement of water out of the ICF (contracted) into the ECF compartment. The water moving into the ECF compartment causes a dilutional hyponatremia, hence the serum sodium is decreased. Since the patient is volume depleted, osmotic diuresis related to glucose is causing the loss of a hypotonic salt solution in the urine, hence the ECF compartment is contracted. Note the unusual situation of an increase in POsm due to hyperglycemia in the presence of hyponatremia. POsm —-PNa® ECF ICF A. High Low — Contracted Contracted Answer: B~ platelets are present in both venous thrombi and in arterial thrombi. In venous thrombi they are only one of the components utilized, the others representing coagulation factors like fibrinogen, V, VIII, and prothrombin. In arterial thrombi, platelets are the primary component of the thrombus. Answer: C~ the patient has fat embolization. Note the time delay before symptoms occur. Answer: E~ metabolic alkalosis Answer: D- mixed: respiratory alkalosis + metabolic acidosis. This could also be a patient with endotoxic shock Answer: F— respiratory alkalosis Answer: B~ metabolic acidosis, ‘- chronic respiratory acidosis. Patient's have COPD. A> acute respiratory acidosis. Note the effect of compensation on respiratory acidosis when comparing COPD (question 7) with this case. Other ABG answers: A = acute respiratory acidosis, B = metabolic acidosis, C = chronic respiratory acidosis Answer: B- excess aldosterone enhances all the aldosterone pumps leading to hypernatremia and hypokalemia, and increased secretion of protons into the urine, leading to retention of bicarbonate and metabolic alkalosis Answer: D- destruction of the adrenal cortex leads to deficiency of aldosterone as well as cortisol and 17-Ketosteroids. Aldosterone deficiency leads to loss of sodium in the urine (hyponatremia), hyperkalemia (cannot exchange potassium for sodium), and retention of protons (metabolic acidosis) Amswer: E- note that the anion gap is 10 mEq/L, which represents a normal AG metabolic acidosis. Adult diarrhea is usually secretory, hence there is an isotonic loss of fluid. Serum sodium levels, therefore, remain normal. There is a considerable amount of potassium (hypokalemia) and nm Note: This material is copyrighted. All rights reserved. (Edward F. Goljan, M.D.) 2001 25. 26. 21, bicarbonate in diartheal fluid. Loss of bicarbonate produces metabolic acidosis. The bicarbonate loss is counterbalanced by an equal gain in chloride ions, which results in a normal anion gap. Answer: F- note that the anion gap is 20 mEq/L, which is an increased AG metabolic acidosis. An inereased AG metabolic acidosis means that the anions of an acid have been added to the ECF— e.g., lactate, salicylate, formate (methyl alcohol), oxalate (ethylene glycol), acetoacetate/p- hydroxybutyrate (ketoacidosis), sulfate/phosphate (renal failure). Answer: A~ as a rule, any serum sodium < 120 mEq/L is SiADH. Note the dilutional effect on all the solutes Answer: C- note the hypokalemic metabolic alkalosis in either of the conditions Nutrition questions 1 10. ML Answer: D~ anorexia nervosa. Amenorthea leaves the patient estrogen deficient and prone to osteoporosis. She should be placed on birth control pills. Note how the stress hormones (cortisol, GH) are increased. Answer: A~ vitamin A deficiency accounts for the first 2 symptoms, while vitamin K deficiency accounts for the hemorrhagic diathesis. Patients with CF have malabsorption and lack all the fat soluble vitamins. Answer: B- vitamin D deficiency causes bone pain and tetany, since the bone is not mineralized properly and is soft (osteomalacia) leaving it prone to fractures. Hypocalcemia explains the tetany, Choice A is scurvy, choice C is pyridoxine deficiency, choice D is thiamine deficiency, choice E is pellagra from niacin deficiency. ‘Answer: E- all the other findings characterize kwashiorkor, where there is a normal total calorie intake but a deficiency of protein Answer: B- com contains niacin which is in a bound form that cannot be absorbed. Choice A is pyridoxine deficiency, choice C is Brz deficiency, choice D is a by-product of niacin, choice E~ maldigestion will cause fat soluble vitamin deficiencies A, D, E, K. Answer: E~ due to a lack of estrogen. All the other findings are more commonly seen in bulimia. Answer: C- increased adipose down-regulates insulin receptor synthesis predisposing the patient to type II DM or glucose intolerance. A urine for free cortisol clearly separates obesity from Cushing's syndrome, the latter having an increase in free levels inthe urine. Answer: E- goat's milk is deficient in folate and pyridoxine Answer: E~ pure vegans do not eat meat or dairy products, hence they are susceptible to Biz deficiency. Vegans who are pregnant should be placed on Biz supplements in addition to the usual prenatal vitamins containing folate and iron. Answer: C- the patient has scurvy. Vitamin C hydroxylates lysine and proline. This is the binding site for the cross-bridges that strengthen collagen by creating a triple helix. Hence, in scurvy, collagen is weakened. Answer: B~ the symptom complex describes acute Wemicke's encephalopathy. Alcohol excess is the MCC of thiamine deficiency. Thiamine is a cofactor for the pyruvate to acetyl CoA reaction using pyruvate dehydrogenase. Giving the patient glucose in an IV causes the rest of the thiamine to be used up, hence precipitating acute Wemicke's encephalopathy. Always give IV thiamine before hanging up glucose. Optional genetics questions L Answer: B- the patient has Down syndrome based on the low serum at-fetoprotein and signs of duodenal atresia. 46 chromosomes indicates a Robertsonian translocation, where the mother had 45 chromosomes, where the 2 chromosome 21s are fused into 1 chromosome. The chromosome from nD Note: This material is copyrighted. All rights reserved. (Edward F. Goljan, M.D.) 2001 the mother essentially has 2 chromosome 21s + the 1 chromosome from daddy leads to 46 chromosomes, however, there are 3 functional chromosome 21s. Nondisjunction is the cause of trisomy 21 ‘Answer: E- since the woman is homozygous for the sickle gene and the man is normal, all the children will have sickle trait Answer: C- testicular feminization. This is a SXR (male) disorder with absent androgen receptors Therefore, fetal testosterone cannot stimulate the development of an epididymis, seminal vesicles, or vas deferens from wolffian duct structures. Similarly, dihydrotestosterone cannot fuse the labia into a scrotum and extend the clitoris into a penis and cannot lead to formation of a prostate gland. Left unstimulated by DHT, the external genitalia remain female in appearance. The lower 2/3rds of the vagina can develop, since it represents the urogenital sinus. All miillerian structures are destroyed by apoptosis, so patients do not have tubes, uterus, cervix, or the upper one-third of the vagina, hence the blind pouch. Answer: D- patient has fragile X syndrome. This is a triplet repeat disorder, hence the disease gets ‘worse in future generations in both the affected males and the female carriers. Some geneticists, therefore call this a sex-linked dominant condition, since female carriers may express the disease. However, other's consider it a sex-linked recessive disorder, which is what the USMLE thinks. Answer: D- microdeletion syndrome on chromosome 15. If the chromosome was maternally derived, the patient develops Angelman's syndrome ("happy puppy" syndrome). If the chromosome is paternally derived, the patient develops Prader Willi syndrome. ‘Answer: C: Sickle cell disease is an AR disease and both parents must carry the abnormal gene, therefore, 1/12 x 1/12 = 1/144 black couples are at risk. They have a 1/4 risk of having a child with sickle cell disease, therefore, 1/144 x 1/4 = 1/576. Hardy-Weinberg equation is the reverse of this in that the prevalence is given and the carrier rate has to be calculated. To simplify the equation, just work backwards. For example, 1/576 + 1/4 = 1/144 couples at risk. V 1/144 = 1/12. Answer: A- urine is the best body fluid to culture CMV and to look for intranuclear inclusions in the renal tubular cells ‘Answer: C mother has rubella. Sensorineural hearing loss is the most common abnormality. Saddle nose deformity refers to congenital syphilis, periventricular calcification to CMV, limb hypoplasia to thalidomide, and craniofacial abnormalities to isotretinoin acid used in treating cystic Neoplasia questions Answers: A, E: adenocarcinoma of the distal esophagus related to Barret's esophagus has replaced squamous cancer of the mid-esophagus as the MC cancer of the esophagus. Adenocarcinoma has also replaced squamous cell cancer as the MC primary lung cancer. ‘Answer: E- p-53 produces a product that inhibits active cyclin D-dependent kinase, which normally phosphorylates the Rb protein and allows the cell to enter the S phase. This allows the cell time to repair defects in DNA before allowing the cell into the S phase. Cells that are damaged beyond repair are removed by apoptosis. Answer: B- all the other sites listed have metastasis more common than a primary cancer. ‘Answer: E~ extranodal metastasis is worse than nodal metastasis. Remember the TNM staging system, which progresses from the least to most important prognostic factor in cancer. ‘Answer: C- since both men and women have colons, it should be no surprise that colon cancer is second in incidence and mortality in both men and women Answer: E- small cell cancers of the lung produce ACTH and ADH Answer: B- these tumor markers are always ordered in testicular cancer. AFP comes from yolk sac ‘tumors (endodermal sinus tumors), while B-hCG comes from choriocarcinoma. Answer: D- patient's will not get HBV, HDV, or hepatocellular carcinoma due to HBV B : This material is copyrighted. All rights reserved. (Edward F. Goljan, M.D.) 2001 Hematology questions L. 10. u. 12. 13. 14. 15, 16. Answer: C- menorrhagia gives away the answer as a cause of iron deficiency in young women. A = thalassemia— note the normal iron studies and high RBC count, B = ACD, D = iron overload~ note the low TIBC (recall that excess iron stores decreases transferrin synthesis), E = acute blood loss initially, the RBC count, Hgb and Het are normal, since whole blood is lost. Plasma is replaced before RBCs, so the RBC deficit is eventually uncovered. Giving the patient isotonic saline immediately uncovers the RBC deficit and the Hgb, Hct, and RBC count will be low. Answer: E~ other differences include achlorhydria (reason for high gastrin levels), autoantibodies against intrinsic factor and parietal cells, correction of Schilling’s test with intrinsic factor. Answers: A, D, E the explanation for A is in question 1. MVO2 is decreased because the patient is in hypovolemic shock and the tissue can extract more oxygen from the blood. The PCWP is decreased because plasma volume is decreased. Answer: A — Streptococcus pneumoniae sepsis is the MC cause of death in children with sickle cell disease. Hence, the importance of Pneumovax and prophylactic antibiotics. Answer: D- the patient has infectious mononucleosis Answer: C- the patient has PRV. The other answers are: A = COPD or cyanotic CHD, B = ectopic EPO production (¢.g., renal adenocarcinoma, squamous cancer of lung), D = relative polycythemia (volume depleted) Answers: A, B, D- the patient has congenital spherocytosis. Increased extravascular hemolysis increases the amount of bilirubin in bile leading to calcium bilirubinate stones and cholecystitis. Answers: B, D, E, G~ they have Pb poisoning with peripheral neuropathy and the abdominal colic. The paint used in pottery often contains Pb. Other scenarios used on boards include eating old paint, working in an automobile factory (incinerate batteries), drinking moonshine (lead lined radiators). Answers: A, C, D- patient has G6PD deficiency (SXR disease) with hemolysis secondary to dapsone. During acute hemolysis enzyme assays are usually normal, since only the cells containing the enzyme remain behind, while those without the enzyme are hemolyzed. Answers: B, C, D, F- the patient has aortic stenosis and a macroangiopathic intravascular hemolytic anemia with schistocytes. Loss of Hgb in the urine has caused iron deficiency. ‘Thrombocytosis is common in chronic iron deficiency. Answer: A~ the patient has sickle cell trait with microinfarctions in the renal medulla, O, tension is low enough in the medulla to induce sickling. Answers: B, C, D~ the patient has B-thalassemia minor. Since B-chain synthesis is decreased, o- chains can combine with 8-chains to produce HgbA; and combine with y-chains to produce HgbF. HgbA is decreased due to the lack of B-chains. Answers: B, C, E-autoimmune hemolytic anemia. Must correct for the anemia and the polychromasia. Type II hypersensitivity reaction. Answers: B, C, E~ the patient has CLL with hypogammaglobulinemia. It is the MC leukemia and ‘cause of generalized lymphadenopathy after 60 yrs of age Answers: A, B, C: the patient has acute lymphoblastic leukemia Answers: A, D, E~ chronic myelogenous leukemia. Blasts do not contain Auer rods (only acute ‘myelogenous leukemia has blasts with Auer rods) Answer: A~ early stage of iron deficiency. Sickle cell trait docs not have anemia, Recall that all the iron studies (iron, TIBC, % saturation, ferritin) are abnormal before there is any anemia. The anemia is first normocytic and then becomes microcytic. 74 Shc NR ne oe ee Note: This material is copyrighted. All rights reserved. (Edward F. Goljan, M.D.) 2001 18. Answer: E- think most common- lack of response to transfusion is most commonly due to a GI bleed. The Hgb should increase by 1 gmi/dL for every unit of packed RBCs and the Het should increase by 3%. 19, Answers: B, D~ patient has acute progranulocytic leukemia with DIC. A positive TRAP stain is hairy cell leukernia and invasion of the gums is acute monocytic leukemia. A low LAP is chronic granulocytic leukemia, 20. Answers: D, E~ remember that iron studies are normal in mild thalassemia (a. or B). Increased RDW is characteristic of iron deficiency. Both have a low MCV. 21, Answers: B, C, E~ patient with SLE and a warm AIHA, which is an extravascular hemolysis leading to an increase in UCB and jaundice. UCB cannot be filtered in the urine like conjugated bilirubin. The reticulocyte index = 4.5% 22, Answers: A, E~ they have no mitochondria and use anaerobic glycolysis for energy 23. Answer: E- decrease adhesion molecule synthesis and destroy lymphocytes and eosinophils 24. Answers: B, C: NOTE- the others are primarily intravascular hemolysis 25. Answers: C, E~ only the Hgb concentration is decrease. Gas exchange is normal, so the PaO; and SaO; should be normal. 26. Answers: C, D- the patient has hairy cell leukemia 27. Answer: C~ these patients frequently progress into an acute myelogenous leukemia. Small and large RBCs and blasts are common in the peripheral blood. Ringed sideroblasts are also a common feature in the bone marrow. 28. Answers: C, D leukoerythroblastic smear from metastasis to bone. Tumor is pushing normal marrow hematopoietic cells into the peripheral blood. 29. Answers: B, E, G, H: NOTE: among parasites, only invasive helminths produce eosinophilia. Pinworms do not invade. 30. Answers: A, C, D, E- the first 3 are bacterial infections due to E. coli. Catecholamines and tissue necrosis explain neutrophilic leukocytosis in an AMI 31. Answers: A, C- parvovirus also produces aplastic anemia in patients with an underlying hemolytic anemia (e.g., sickle cell disease, congenital spherocytosis) 32. Amswer: B- most microcytic anemias in children are due to a bleeding Meckel's diverticulum 33. Answer: B- most microcytic anemias in adults over 50 are due to colon cancer 34. Answer: B- somehow, it increases vessel permeability leading to cerebral edema 35. Answers: A, B, C NOTE: the neurologic deficits remain, hence the importance of making the correct diagnosis, 36. Answers: A,B HgbA is never present in sickle cell disease 37. Answer: C— in children without sickle cell disease, Staphylococcus aureus is the MCC of osteomyelitis, not Salmonella 38. Answer: B: the abnormal chromosome has (1) 1.35 kb segment, while the normal chromosome is cleaved into (1) 1.15 kb fragment and (1) 0.2 kb fragment. Patient A has sickle cell disease, with (2) uncleaved 1.35 kb segments. patient C is normal, and both 1.35 kb segments have been cleaved into (2) 1.15 kb fragments and (2) 0.2 kb fragments Lymphoproliferative questions 1. Answers: A, D, E: the patient has multiple myeloma. Note the urine findings that screen for BY protein (disparity between the dipstick for protein and SSA, the latter detecting both albumin and globulins, while the former only detects albumin). Tubular casts in the urine imply that acute tubular necrosis has already occurred. 2. Answers: A, E: an IVP would further exacerbate the patient's renal disease. IEP identifies the abnormal Ig and light chain, 3. Answer: D: B cell malignancy with a t(8;14) of the e-myc proto-oncogene 5 Note: This material is copyrighted. All rights reserved. (Edward F. Goljan, M.D.) 2001 8 ‘Answer: B- Sezary syndrome. It is called mycosis fungoides if the cells are not in the blood. Answer: A~Hand-Schuller-Christian disease. CD1 is a histiocyte marker. Answer: F- Waldenstrom's macroglobulinemia an IgM monoclonal spike must have been present, Multiple myeloma does not usually have an IgM spike. Answer: C: nodular sclerosing Hodgkin's disease. This is the classic presentation of nodular sclerosing HD in women or men ‘Answer: E: these are all features of amyloidosis Coagulation questions 10. ML. 12. 13, 14. Answer: B- the patient has von Willebrand's disease Answer: D- the patient is most likely taking aspirin or other type of NSAID. The only Rx that could correct this problem would be a platelet transfusion, which is certainly not indicated in this case but would be in a life-threatening bleed. Answers: B, C, E, H: the patient has DIC and is in renal failure (BUN/ereatinine ratio <15) Answer: E: in DIC, the most effective Rx is to Rx the cause of the DIC, in this case, endotoxic shock secondary to E. coli. The blood components are also indicated, but essentially feed the fire and keep the patient alive until the underlying disease is eradicated, Answers: C, E, F- good comparison question Answer: E- rat poison is warfarin, which blocks all the vitamin K-dependent factors. Both PT and PTT are prolonged Answer: E- heparin enhances ATI, which neutralizes most intrinsic factors and most of the final common pathway factors, hence both PT and PTT are prolonged Answer: B: the patient has DIC and is consuming platelets, fibrinogen, prothrombin, V, VIII Answer: A: HUS due to 0157: H7 serotype of £. coli. Remember that only platelets are consumed, not coagulation factors Answer: D: hemophilia A, maternal father transmits the disease to all his daughter's (SXR trait) who are asymptomatic carriers. The daughters transmit the gene to 50% of their sons, Answer: A~ the patient has idiopathic thrombocytopenic purpura Answer: A~ the patient has thrombotic thrombocytopenic purpura Answer: C: the patient has hereditary ATI deficiency. Heparin cannot anticoagulate without ATIIL The pearl is the lack of increase in the PTT with heparin. Answer: D: heterozygote carriers for protein C become homozygote when given warfarin in ~6-8 hrs when the half-life of previously y-carboxylated protein C disappears. Now the patient is hypercoagulable. Blood Bank questions 1 2 Answers: C, D, F: the patient has had a delayed HTR due to an antibody directed against an antigen on the donor RBCs. Answers: A, B, C, E: ABO incompatibility does protect against Rh sensitization, but she still should receive Rh immune globulin. Mother's anti AB IgG is coating the babies RBCs. The Apt test distinguishes fetal from adult Hgb. The baby has swallowed mommies blood and does not have aGIbleed. Answers: C, E: febrile reaction: patient must have been exposed to blood products at some time in her life for antibodies to develop (common in multiparous women who commonly have fetomaternal bleeds during delivery) A; elderly people lose their isohemagglutinins, hence they may not develop hemolytic TRs even with an ABO mismatch 16 Note: This material is copyrighted. All rights reser ed, (Edware 1. Gay 10. ue 12, 13. 14. 15. 16. “Amswers: A, D: mother's get sensitized against Rh antigen 12 their first Rh incompatible pregnancy, Au this does not affect that baby but future babies “Answers: C, D: allergic reaction ‘Answer: E: the mother is ABO and Rh incompatible with her baby. She has not been previously Anstfiaed to D antigen, soit cannot explain the babys Yaundice, therefore, ABO hemolytic disease df the newborn is the most likely cause ‘Answers: B, D, F- a major crossmatch involves mixing patient serum with donor RBCS to see if they are compatible. rey wer. D: blood group © people can only receive blood group ©. Remember that the forward type with antisera identifies the ‘blood group, while the back type using test RBCs, identifies the ‘antibody associated with the blood group Answer: B: she must ‘be AB to have A, B, or AB children. AB people are universal recipients, since they lack antibodies. vanswer: A: gastric adenocarcinomas are usually seen in group A patients. Tt is always best to give the patient the same ABO group rather than O packed RBCs. ‘Answer: C: the patient ‘must be blood group B ‘A: 1:300 chance of becoming HIV positive sae: HBV has the greatest viral oad in blood of all viruses. You cannot get syphilis from transfused blood. rans D. itis almost impossible to find blood that is negaive for antibodies against CMV Answer: C: HCV is the MCC of transfusion hepatitis Cardiovascular questions 1 2. 10. u 12, ‘Answer: A~ ruptured abdominal aortic aneurysm. Note the rupture triad of left flank pain, ‘hypotension, and pulsatile mass. Rpewer: B~ dissecting aortic aneurysm. Note the bye of pain radiation into the back, absent pulse, nsryur of aortic regurgitation and widening ofthe aor knob. ‘Answer: D~ mitral valve prolapse. When the LVEDV is decreased (standing, anxiety) the click and Atemur come closer to SI. When the LVEDV is Tperoased (lying down, clenching fist), the click ‘and murmur come closer to S2. sewer, B~ hypertrophic cardiomyopathy. MCC of sudden death in young people. The 30 mm Hg. Any lung disease would be expected to increase the gradient- e.g., pneumonia, COPD, sarcoidosis, ARDS. Hypoxemia due to conditions not directly involving the fungs have a normal gradient, For example, you would expect a normal A-a gradient in paralysis of the diaphragms, barbiturate overdose, and obstructive sleep apnea due to enlarged tonsils. Gastrointestinal questions 1. Answer: G~ this is due to S. aureus producing a toxin which is ingested in the food. Culture the food, not the stool ‘Answer: E~ Campylobacter is the MCC of invasive gastroenteritis in the U.S. It can produce erypt abscesses and simulate UC. 3, Answer: F- Yersinia is a cause of mesenteric adenitis, a granulomatous disorder, that can be confused with acute appendicitis. 4, Answer: A- Vibrio cholerae produces a secretory diarrhea due to toxin stimulation of c-AMP. 5, Answer: D~ Salmonella typhi produces this classic triad during the second week of infection. A human reservoir is involved. ‘Answer: B- B. cereus produces a preformed toxin. Salmonella enteritidis is the MCC of food poisoning ‘Answer: D— Cryptosporidium, a sporozoan, is the MC pathogen in AIDS diarrhea ‘Answer: C- it isa secretory diarrhea due to a heat stable toxin that stimulates guanylate cyclase 9. Answer: B- Candida is the overall MC fungus infection in AIDS. Esophagitis is AIDS-defining, but not thrush, 10, Answer: A- irritable bowel syndrome IL. Answer: C- both are associated with H. pylori. Duodenal ulcers have the greatest association with . pylori and also have the highest incidence of perforation and an association with ZE in MENT 12. Answer: B- self explanatory 13. Answer: B- all the other choices characterize CD 14. Answer: C- lactase deficiency. Lactase is a disaccharidase and a brush border enzyme. It produces an osmotic type of diarrhea. 15. Answer: E- there is also evidence of liver metastasis, 16. Answer: E- the patient has the carcinoid syndrome. Appendiceal carcinoids do not metastasize to the liver, so it would not produce the syndrome. 17. Answer: E~ the patient has angiodysplasia. There is a relationship with aortic stenosis and von Willebrand's disease. Diverticulosis is the MCC of hematochezia, however, the barium enema is normal. 18, Answer: D- acute diverticulitis presents as a "left-sided appendicitis" 19. Answer: A~ note Virchows node and the history of weight loss. Remember that a postmenopausal Qworan should not have palpable ovaries. It could by primary ovarian cancer, or as in this case, ‘metastasis to the ovaries (Krukenberg tumor) from stomach cancer. 20. Answer: E~ atrial fibrillation is the MC arrhythmia associated with embolization, in this case, to the superior mesenteric artery. Amylase is present in the small bowel, so do not be fooled into calling this hemorrhagic pancreatitis. 21. Answers: C, D- the patient has achalasia 7 8 9 ‘Note: This material is copyrighted. All rights reserved. (Edward F. Goljan, M.D.) 2001 Hepatobiliary/pancreas questions |. Answer: B- varicella or influenza infections + salicylates may lead to Reye's syndrome 2. Answer: C— A= serologic gap: not infective since HBeAg and HBV DNA are absent, B= acute chronic HBV, D = vaccinated: note the difference from someone who has recovered~ there is anti-HBe-IgG 3. Answer: A~ Rx is metronidazole 4. Answer: D- if the PT had corrected, then the patient has vitamin K deficiency, which, in t Patient, could be due to malabsorption of K from chronic pancreatitis or bile salt deficiency rela: to his cirrhosis 5. Answer: C~ the patient has hemochromatosis, an AR disease with increased iron absorption fre the small bowel. The term "bronze diabetes" describes the skin pigmentation and DM. Iron in tise generates hydroxyl FRs which damage the tissue~ cirrhosis, chronic pancreatitis, restrict cardiomyopathy. Serum ferritin is the best screening test. Hepatocellular carcinoma is the \ cause of death. 6 Answer: D- CB >50% always indicates obstructive jaundice. Choice A (Gilbert's) would have CB <20% (decreased uptake and conjugation), choice B would have a CB 20-50% (problem wi upiake/conjugation and necrosis of bile ducts causing release of CB), choice C (Crigler Naja ‘would have a CB <20% (deficiency of conjugation enzymes), choice E (EHA) would have « C “<20%, due to macrophage destruction of RBCs and UCB as the end-product of Hgb degradation. % Answer: E- note the obstructive signs of light colored stool and palpable GB (Courvoisie’s sign) 8. Answers: D, E- the patient has Wilson's disease. Note that total copper is low beeaus ceruloplasmin, the binding protein for copper is low. The Kayser-Fleischer ring is described. 9. Answers: B, C, E- the patient has primary biliary cirthosis, where there is granulomato destruction of bile ducts in the portal triads. IgM levels are increased as well as cholesterol due t obstruction to bile. 10. Answers: A, C, E~ gynecomastia is due to estrogen excess and encephalopathy due to an increas in ammonia and false neurotransmitters 11. Answer: E- birth control pills produce intrahepatic cholestasis 12, Answer: B- similar to question 11, anabolic steroids also produce intrahepatic cholestasis 13. Answer: B- this is the classic history for HCC. Note the increase in AFP. 14. Answer: D- the patient has echinococcosis. The cycle for the parasite is egg to larva to adult. The host with the larva is always the intermediate host (infected sheep, sheepherder), while the hos with the adults is the definitive host (dog who ate the infected sheep). The sheepherder got the disease from the dog who has the adults that produce eggs. The eggs develop into the larve in the sheepherder, who is the intermediate host. Basque's and Greek’s are famous sheepherders, 15. Answers: A, C— always assume that weightlifters, wrestlers, or professional football players are or Steroids. Anabolic steroids and estrogen not only produce intrahepatic cholestasis but also produce benign tumors called liver cell adenomas. They have a tendency to rupture and produce intraperitoneal hemorrhage, 16. Answer: B- the patient is an aleoholic (AST > ALT) with acute pancreatitis (pain radiating into back and sentinel loop due to localized ileus). Renal questions 1, Answer: C- the patient has metastatic prostate cancer. The best first test is a rectal, which would definitely identify cancer. The other tests can then be ordered. 2. Answer: B- the key is the cryptorchidism. Even the uninvolved testicle is at risk. 80 Note: This material is copyrighted. All rights reserved. (Edward F. Goljan, M.D.) 2001 13, rr Answer: B~ note the smoking history. Both lung and renal cancer are associated with smoking. In this case, hematuria and the flank mass indicates the kidney as the source. Renal metastasis is uncommon. Answer: B- the patient has acute pyelonephritis (fever, WBC casts, flank pain). This is due to ascending infection and vesicoureteral reflux ‘Answer: B- the patient has Goodpasture's syndrome. Note how patients usually start the disease in the lungs and then the renal disease occurs. Anti-glomerular and pulmonary capillary basement membrane antibodies are responsible. Rapidly progressive crescentic GN is the most common type of renal disease. Answer: B~ association of lung cancer with membranous GN, which is the MC type of GN causing the nephrotic syndrome in adults Answer: C- focal segmental glomerulosclerosis is the MC renal disease in IVDA and AIDS. It is a severe type of nephrotic syndrome. Answer: E~ pubertal gynecomastia. Recall that gynecomastia is normal at birth, puberty, and in old age. It relates to hyperestrinism, Answer: A~ note the history of hypertension and a unilateral abdominal mass Answer: A~ the patient has lipoid nephrosis Answer: D- the patient has ischemic ATN. BUN/creatinine ratio should be <15/1 (c.g., 80/8), UOsm <350 mOsmvkg (no concentrating ability), and the random UNa > 40 mEq/L (tubular dysfunction) Answer: A~ APKD is an AD disorder, hence the family history. Mitral valve prolapse is a common association including intracerebral bleeds and subarachnoid hemorrhages from ruptured berry aneurysms, Answer: C- cyclophosphamide is the Rx of choice for WG. It produces both hemorrhagic cystitis and transitional cell carcinoma. Answer: C- this is Klinefelters syndrome Gynecology questions 10. i 12. ‘Answer: D- these develop in the lactiferous duct. Answer: A~ infiltrating ductal carcinoma is the MC primary breast cancer. Lobular cancer if the ‘MC cancer that is bilateral Answer: C- FCC is the MCC of a mass in the breast in a woman under 50 yrs old. Note that it is painful, while cancer is painless. Answer: F— unlike Paget's disease of the vulvar, Paget's disease of the breast is an extension of a ductal cancer into the epithelium of the nipple. Answer: B- first generation relatives (mother or sister) overrides age Answer: D~ fibroadenomas are stromal tumors that compress ducts. They are the MC tumor in ‘women <35 yrs old Answer: A~ a similar presentation in the first trimester would be a molar pregnancy Answer: D- cystic teratoma Answer: C- in her age bracket, induration in the pouch of Douglas is due to seeding from a primary ovarian cancer. CA125 is a tumor marker for surface derived ovarian cancers (e.g., serous cystadenocarcinoma). Answer: D- preeclampsia in the first trimester is a molar pregnancy. Answer: C- cervical cancer is least common owing to detecting dysplasia on a Pap smear and treating it before it develops into cancer Answer: C— cervical cancer infiltrates into the lateral wall of the vagina and blocks the ureterovesical junction causing hydronephrosis/pyelonephritis and death. 81 ‘Note: This material is copyrighted. All rights reserved. (Edward F. Goljan, M.D.) 2001 13, 14, 15. 16. 17, Answer: C- this is as classic a presentation as it gets ‘Answer: D- early menarche, late menopause, nulliparity, obesity, type Il DM are all increased risks for endometrial cancer, which relates to prolonged exposure to unopposed estrogen ‘Answer: D~ early sexual exposure, multiple high risk sex partners, smoking, birth control pills are risk factors ‘Answer: C- the patient has classic endometriosis ‘Answer: C- the patient has Turner's syndrome, the MC genetic cause of primary amenorrhea, Endocrine questions 10, i 12. ‘Answer: C- patient is taking excess thyroid hormone. Note the low 1] It would be high in Graves disease. ‘Answer: B- estrogen effect on increasing TBG without altering the free hormone level. Mitral valve prolapse is an incidental finding ‘Answer: E— primary hypothyroidism A = Graves disease, D = Androgen effect ‘Answer: C- renal stone in primary HPTH. Answer: A- DiGeorge syndrome, which has absence of the thymus and parathyroid glands from failure of development of the 3rd and 4th pharyngeal pouches. Choice A could also be primary hhypoparathyroidism. B = secondary hyperparathyroidism, D = malignancy induced and all other types of hypercalcemia ‘Answer: A~ metyrapone blocks 11 hydroxylase, so ACTH and 11-deoxycortisol should be increased if the pituitary and adrenal cortex are ‘normal. In hypopituitarism, neither ACTH or deoxycortisol is increased. In Addison's disease, ACTH is increased, but deoxycortisol is decreased. Choices B and C occur in neither condition. The 17 ketosteroids (DHEA and androstenedione) would be low in both of them. Both would have hyponatremia. In hypopituitarism, the loss of ‘cortisol and thyroid hormone leaves ADH unchecked, therefore there is a mild hyponatremia. Remember that ACTH has no effect on aldosterone release. In Addison's, there is @ loss of aldosterone, hence hyponatremia is much more severe. Choice D is only present in Addison's and choice E is only present in panhypopituitarisr Answer: B— when insulin is absent, fatty acids undergo B-oxidation. Choice A occurs in the presence of insulin and is inhibited in DKA. Choice C occurs when insulin is present in order to synthesize fatty acids or when it is absent (DKA) due to increased f-oxidation of FAs. Choice D cecurs when insulin is present and is inhibited in DKA. Choice E occurs when insulin is present. ‘The enzyme is activated in DKA due to glucagon and catecholamines. ‘Answer: A~ the patient has a prolactinoma. Prolactin inhibits GnRH, hence the amenorrhea. Since the TSH is normal, primary hypothyroidism cannot be the cause. ‘Answer: D- in pituitary Cushing's, cortisol is suppressed with the high dose dexamethasone test and not in other types of Cushings. All the other choices are the same in all of them. ‘Answer: A~ the high C-peptide proves that the patient has an insulinoma. It would be suppressed if the patient was surreptitiously taking human insulin, since it would suppress endogenous insulin release. ‘Answer: A~ osmotic damage of Schwann cells produces a sensorimotor peripheral neuropathy. Glucose is converted by aldose reductase into sorbitol, which is osmotically active. ‘Answer: B- in patient's with either central or nephrogenic DI, water deprivation results in a low Uosm (ADH cannot concentrate the urine) and a high POsm, which correlates with bypernatremia. When vasopressin is given to the patient, those with central DI will have a rise in UOsm >50% from the baseline, while those with nephrogenic DI have less of a response (choice A). Patients with psychogenic polydipsia behave like a normal control, in that they can concentrate urine with water deprivation. 82 BROMIDE D THES SHSTSOSOSSSSSECETSSOEVELCELEEE Note: 1B. 14, 15. 16, 17. 18. 19. 20, 21 22. ‘This material is copyrighted. All rights reserved. (Edward F. Goljan, M.D.) 2001 “Answer: A~ insulitis is due to autoimmune destruction of the B-cells. In type II DM, the islets are fibrosed and often contain amyloid. Choices B and D are associated with type II DM. Choices C and E occur in both types. ‘answer: A~ the child has a craniopharyngioma with optic chiasm compression and hypopituitarism. IGF-1 would be low. Hypokalemia, low urine 17-OH levels (cortisol and deoxycortisol), low serum TSH, and low serum deoxycortisol after metyrapone would be expected. ‘Answer: B~ the patient has pheochromocytoma. The history is classic. Metanephrines are more sensitive than VMA as a screen. ‘Answer: F- the child has a neuroblastoma with metastasis. Neuroblastomas are APUD tumors. ‘They derive from neural crest, have neurosecretory granules, and are $100 antigen positive. Other ‘examples are small cell cancers, carcinoid tumors, and melanomas. ‘Answer: D- gluconeogenesis is the key reaction that produces hyperglycemia in DKA, therefore the alanine to pyruvate to glucose is the best answer ‘Answer: C- the patient has Sheehan's postpartum necrosis. Prolactin is the first hormone to decrease, hence cessation of lactation. ‘Answer: E- these are the symptoms of hypoglycemia from too much insulin. A candy bar will suffice. ‘Answer: B~ the patient has MEN Ila with medullary carcinoma (calcitonin is the tumor marker), hnyperparathyroidism, and pheochromocytoma. ‘Answer: A~ since the seminiferous tubules have Sertoli cells that synthesize inhibin, which is the negative feedback for FSH, the FSH would be increased. Leydig cells synthesize testosterone, ‘which has a negative feedback with LH, so both of these tests would be normal. The sperm count is tow, since they develop in the seminiferous tubules. Choice B is Leydig cell dysfunction, choice C is both seminiferous and Leydig cell dysfunction. ‘Answer: C- primary aldosteronism. Aside from Cushing's syndrome, no other disorder has this type of electrolyte profile + hypertension. Tetany is due to metabolic alkalosis. An alkaline pH increases the number of negative charges on albumin, hence more of the free ionized calcium can be bound to albumin. The total calcium remains the same, but the ionized calcium is decreased, hence the presence of tetany. ‘Musculoskeletal and autoimmune questions ‘Answer: C- the patient has gout involving the big toe (podagra). It is most commonly due to lunderexcretion rather than overproduction of uric acid. The crystals are negatively birefringent, ‘meaning that they are yellow when parallel to the slow ray of the compensator. ‘Answer: D- joint fusion due to pannus destruction of the articular cartilage is a characteristic of RA. OA is not an inflammatory arthritis. All the other choices are findings in OA. ‘Answer: B- the patient has classic Lyme disease, due to the spirochete Borrelia burgdorferi). Its transmitted by the Ixodes tick. The rash is erythema chronicum migrans. Rx is doxycycline. ‘Answer: D~ the patient has disseminated gonococcemia due to Neisseria gonorrhoeae, a gram negative diplococcus. Some of these patients are deficiency in complement components C5-C9. ‘Answer: D- cat bites are more likely to cause this than dog bites. Penicillin is the Rx of choice. ‘Answer: C- sacroiliitis and male dominance (HLA B27 positive) characterize AS. Pannus and theumatoid factor (IgM antibodies against IgG) characterize RA. ‘Answer: E~ the patient has ankylosing spondylitis and uveitis ‘Answer: D- the patient has ulcerative colitis which in an HLA B27 positive individual is the trigger for producing ankylosing spondylitis. ‘Answer: C- the patient has juvenile rheumatoid arthritis, specifically Still's disease. 83, ‘Note: This material is copyrighted. All rights reserved. (Edward F. Goljan, M.D.) 2001 10. ML. 12. 13. 14, 15. 16. 17. Answer: C- both lose articular cartilage, but for different reasons. In OA, it is wom down by weight and ischemia, while in RA, it is destroyed by pannus. Only RA affects the MCP joint. Subchondral bone cysts are a feature of OA. Answer: B- increased destruction of cells leads to increased metabolism of purines into uric acid leading to urate nephropathy. The patient should have been given allopurinol. Answer: B- the patient has tabes dorsalis and a neuropathic (Charcot's) joint. The patient is unable to sense pain. Answer: E~ the symptom/sign complex characterizes rheumatoid arthritis. It describes associations with pneumoconiosis (Caplan's syndrome), Sjogren's syndrome (dry eyes and dry mouth, due to autoimmune destruction of the lacrimal glands and minor salivary glands), and Felty’s syndrome (splenomegaly with neutropenia). Answer: C- the patient has drug-induced lupus, with hydralazine as the offending agent. Procainamide is another drug. Answer: C- the patient has SLE with a classic pericardial effusion. Answer: C- sclerodactyly and dysphagia for solids/liquids is seen in PSS and CREST syndrome. A. false positive syphilis serology due to anti-cardiolipin antibodies is seen in SLE as well as Libman- Sacks endocarditis. Answers: C,F,H,1,J A. Osteomyelitis Most common cause in sickle cell disease is Staphylococcus aureus: no, Salmonella species are the MCC B. Osteomyelitis. Most common site is the metaphysis. Most common mechanism for spread to bone in children is local extension from a soft tissue abscess: no, the MC spread is hematogenous to the metaphysis. . Chronic osteomyelitis Orifice of sinus tract is subject to developing squamous cell carcinoma: yes, this is also true for third degree burn sites. D. —_Pott's disease Staphylococcus aureus involving the vertebral column: no, it refers to TB of the vertebral bodies Lyme's disease Most common late manifestation is arthritis. Bilateral Bell's palsy may occur: yes, and bilateral Bell's palsy is almost pathognomonic of Lyme's disease. G. —_Osteopetrosis AD disorder with an osteoblastic defect. Too little bone. Pathologic fractures, deafness: no, it is am osteoclastic defect, so there is too much bone (brittle bone disease) and no bone marrow. Osteogenesis imperfecta AD disorder with a defect in synthesis of type 1 collagen. Blue sclera due to underlying choroidal veins. Pathologic fractures: yes, this is a favorite question and slide. L Osteoporosis ‘Most common cause in women is estrogen lack. More bone is lost by osteoclastic activity than replaced by osteoblastic activity: yes, the lack of estrogen leaves interleukin-1 unchecked, so osteoclastic activity overrides osteoblastic activity reducing overall bone mass. & 4, Osteoporosis Most common fractures are vertebral compression fractures: yes, followed by Colle's fracture K. Osteoporosis. Routine bone x-rays are the most accurate method for diagnosing the condition: no, it only shows osteopenia. Dual beam absorptiometry is the diagnostic test of choice. Rx is estrogen, vitamin D, calcium, and stress-types of exercise (walking, weight lifting, not swimming) 84 Note: This material is copyrighted. All rights reserved. (Edware F. Golan, Mads) 4vv 18. Answers: A, B,D, E, F A 19. ‘Benefits of estrogen in Maintains high HDL and low LDL. Increases bone density osteoporosis rather than preventing loss of bone density. Danger of endometrial cancer if progesterone is not added. Femoral neck fracture Danger of aseptic necrosis from compromise of medial femoral circumflex artery by a posterior dislocation: yes, this is the MCC of aseptic necrosis of the femoral head followed by corticosteroid therapy. Scaphoid bone fracture Danger of osteomyelitis. Pain in the anatomical snuff box: Colles’ fracture no, the danger is aseptic necrosis, but yes, it does produce pain in the anatomic snuff box. Dinner fork deformity of the distal radius. Common fracture when falling on the outstretched hand. Supracondylar fracture ‘Fracture of the distal humerus. Median nerve may be injured Aseptic necrosis Legg-Perthes Brachial artery compromised leading to possible ischemic contracture of forearm muscles: this is commonly asked on boards. Bone x-ray reveals increased density. MRI is the most sensitive test: yes, the increased density is due to reactive bone formation. Osteomyelitis of the femoral head in children < 10 yrs old: no, it is aseptic necrosis of the femoral head in this age bracket. Osgood-Schlatter’s disease Inflammation of the proximal tibial apophysis with repair Answers: A, B,D, E,G & im IP ™ Paget's disease ‘bone Osteochondroma Ewing's sarcoma ‘Chondrosareoma Osteoma resulting in knobby knees. Leads to problems with bone growth in the leg: yes, for the former sentence, but no, it does not affect bone growth in the leg like Legg-Perthes disease. of Bone thickens and fractures easily. Serum alkaline phosphatase is elevated, Complications include osteogenic sarcoma, pathologic fractures, high output cardiac failure: yes, it primarily affects elderly males. Lytic lesions are noted in bone and may be ‘confused with multiple myeloma except they have irregular and fuzzy margins as opposed to myeloma, which has sharp, cookie cutter type margins. ‘Most common overall benign bone tumor: yes, it is an exostosis of bone that is capped by benign cartilage. "Round (small) cell tumor" with x-ray findings of “onion skinning”, Primarily affects young adults: yes for the type of tumor and the x-ray findings, but no, it is primarily = childhood tumor. Most common malignant cartilaginous tumor. Grade determines it Ddiologic behavior. Most commonly affects pelvic bones. Radiograph demonstrates a radiolucent defect surrounded by densely sclerotic bone. Nocturnal pain is relieved by aspirin Osteogenic sarcoma Association with Rb suppressor gene on chromosome 13. “Sunburst appearance" and "Codman's triangle" on x-ray. Primarily affects the pelvic bones: yes for everything except it primarily involves the knee area— lower femur or upper tibia 85 — ETT ————————————————————_——_—_————_, ‘Note: This material is copyrighted. All rights reserved. (Edward F. Goljan, M.D.) 2001 in the metaphyseal area. G. Muscle weakness Causes include upper/lower motor neuron disease, primary muscle disease, myasthenia gravis, and extrapyramidal disease H. — Dupuytren's Fibromatosis involving palmar tendon sheaths. Related to contracture hyperestrinism in cirrhosis: yes for its location and classification but no for any relationship with hyperestrinism. L Desmoid tumor Fibrosarcoma of the abdominal wall often associated with Turcot's syndrome: no, it is 2 fibromatosis often associated with Gardner's syndrome 20. Answers: C,D ‘A. Embryonal rhabdomyosarcoma Most common adult striated muscle sarcoma. Presents as a vaginal mass in women or bladder/prostate mass in men: yes for everything except it is the most common childhood sarcoma. B. —_Lipoma ‘Most common benign soft tissue tumor in women: yes for men but a leiomyoma of the uterus is the most common soft tissue tumor in women. Gc Leiomyoma ‘Most common benign soft tissue tumor of the uterus and gastrointestinal tract: yes, and in the GI tract it is most common in the stomach, where it is associated with bleeding. D. —_Rhabdomyomas Most common benign tumor of the heart in children with tuberous sclerosis: yes, cardiac myxoma ‘is more common for adults while cardiac rhabdomyomas is more common children E. ‘Malignant fibrous histiocytoma Most common childhood soft tissue sarcoma. Associated with radiation exposure and scars: yes for everything, except it is the most common adult sarcoma followed closely by liposarcoma. ‘Skin questions 1 Answer: B- ACE is also involved in the metabolism of bradykinin. Hence, an ACE inhibitor leads fo am increase in bradykinin (increases vessel permeability), which is the mechanism for angioedema and cough. “Answer: A~ T. tonsurans invades the inner hair shaft, hence the fluorescent ‘metabolites cannot be detected by Wood's lamp light (UVA_ light). It is now the MCC of tinea capitis. M. canis used to be the MC cause of tinea capitis. It is Woods lamp positive. 3, Answer: C- procainamide is another drug that is associated with drug-induced SLE, Anti-histone antibodies are pathognomonic of drug induced lupus. 4. Answer: E- the patient has porphyria cutanea tarda. 5. Answer: E~ the patient has acute intermittent porphyria & Amswer: C-note thatthe distribution of the rash isin sun-exposed areas, soa drug associated with photosensitivity is likely. 7, ‘Answer: E- the patient has condyloma acuminata, which is due to HPV 6 or 11. Topical podophyllin is the Rx of choice. Do not confuse this with condyloma latum, which is secondary syphilis. 4g, ‘Answer: B- the patient has psoriasis, In blacks, the erythema ofthe lesion is masked, however, 3 Silvery scales are present. Nikolsky sign is noted in pemphigus vulgaris, @ bullous lesion with 86 Note: This material is copyrighted. All rights reserved. (Edward F. Goljan, M.D.) 2001 10. ML 12, 13. 14. 15, 16. V7. 18, 19. 20, separation above the basal cell layer. Dermatographism is noted in chronic urticaria. Wickham's stra isa fine, lacy leukoplakic lesion on the buccal mucosa in lichen planus. ‘Answer: B- the Breslow system measures depth of invasion. Lesion > 1.7 mm in depth have the capacity to metastasize. ‘Answer: B- the patient has atopic dermatitis. Atopy is a characteristic feature in the family. ‘Answer: D- the patient has a type IV contact dermatitis to nickel, which is similar to the pathogenesis of poison ivy and a positive PPD. Choice A and B are a type I reaction involving histamine. Choice C is type I immunocomplex disease. Tinea corporis is a superficial dermatophyte infection, most often due to Trichophyton rubrum. ‘Answer: B~ the patient has tinea corporis, which is most often due to Trichophyton rubrum. ‘Answer: A~ the patient has molluscum contagiosum, which is a DNA poxvirus ‘Answer: B- the patient has actinic (solar) keratosis, a UVB related precursor lesion for squamous cell carcinoma, ‘Answer: B~ the patient has tinea versicolor, which is due to Malassezia furfur, as superficial dermatophyte that also causes seborrheic dermatitis (dandruff) ‘Answer: C- the initial rash is called a herald patch. The rash distribution is described as being in a "Christmas tree" distribution. It is best treated with UVB light. Answer: B~ the patient has chickenpox, which is a DNA Herpesvirus in the same family has HSV, CMY, and EBV. The inclusions of HSV and varicella look the same. Note the different stages of the rash, which is characteristic of varicella. ‘Answer: A~ the patient has shingles. The virus is in the sensory ganglia in the thoracic area. ‘Answer: D- the patient had impetigo due to a nephritogenic strain of group A streptococcus which lead to post-streptococcal glomerulonephritis. Answer: D- the patient has acne vulgaris, CNS and special senses questions 1 10. ul ‘Answer: D- all of the S/S are due to thiamine deficiency and Wernicke's encephalopathy. Recall that the mamillary bodies and tissue around the ventricles have an orange discoloration due to hemorrhages and subsequent deposition of hemosiderin. ‘Answer: B~ the patient has Wilson's disease. Degeneration occurs in the lenticular nuclei. If it occurs in the subthalamic nuclei, hemiballismus may occur. ‘Answer: D- the patient has Creutzfeldt-Jakob disease ‘Answer: D (Neisseria meningitidis). If the patient was over 18 yrs old, the answer would be A (Streptococcus pneumoniae) Answer: B- this is the MC primary malignant CNS tumor in children, but a cystic astrocytoma of the cerebellum (benign) is the MC overall primary CNS tumor. ‘Answer: D- the patient has metastatic lung cancer to the brain with uncal herniation and compression of the oculomotor nerve leading to mydriasis, lid-lag, and ophthalmoplegia (eye deviated down and out) Answer; A- the patient has Down syndrome. The extra chromosome 21 codes for more B-amyloid protein, hence Alzheimer's disease at an early age ‘Answer: A~ MS is the MC demyelinating disease. It is an autoimmune destruction of the myelin sheath. Recall that bilateral internuclear ophthalmoplegia is pathognomonic of MS. ‘Answer: E- sensory changes are due to VIIth nerve tumor in cerebellopontine angle impinging on the Vth CN. There is an association of acoustic neuromas with neurofibromatosis. Answer: C- atrial fibrillation is the MC arrhythmia leading to embolic disease. This is a hemorrhagic infarction (liquefactive necrosis), while atherosclerotic strokes are pale infarctions. ‘Answer: D- this is due to deficiency of dopamine in substantia nigra neurons 87 Note: This material is copyrighted. All rights reserved. (Edward F. Goljan, M.D.) 2001 12, 13, 14. 15, 16. 17, 18. 19, 20. 21 22. 23 24. 25, 26. 28. 29, 30. 31. 32 33. 34, 35, Answer: A~ classic question between syringomyelia and ALS. ALS has no sensory changes, while syringomyelia does along with motor changes. ‘Answe?: C- toxoplasmosis is the MCC of a space occupying lesion in the brain in AIDS. Note the low helper cell count. Toxo is found in cat liter and in undercooked meat. Answer: E~ it is the second MCC of neonatal meningitis/sepsis, with group B streptococcus (S. agalactiae) the MC. All the other disorders characterize Pseudomonas C- the patient has tabes dorsalis (neurosyphilis). Note the Argyll-Robertson pupil. E- the patient has neurofibromatosis with neurofibromas and café au lait spots. B- the patient has cryptococcus. CSF protein is elevated and glucose is low. D- the patient has rhinocerebral mucormycosis (C- the patient had an epidural hematoma ‘Answer: D~ the patient had a subdural hematoma with rupture of the bridging veins between the arachnoid and dura ‘Answer: E- the patient had a hypertensive bleed ‘Answer: A~ the patient had a subarachnoid hemorthage from rupture of a congenital berry aneurysm, which developed over time due to hypertension secondary to APKD. Answer: B- the patient has 2 glioblastoma multiforme, the MC primary brain tumor in adults. ‘Answer: E- the patient had a meningioma, which is the most common benign primary brain tumor in adults. It is a common cause of focal new onset seizures. Psammoma bodies are present in the tumor. ‘Answer: B- papilledema implies an increase in intracranial pressure Answer: B- the virus itself is responsible for the cancer. ‘Answer: C- the patient has Guillain-Barre disease, an autoimmune disease with destruction of ‘myelin in the peripheral nervous system Answer: C- the patient has Meniere's disease with sensorineural hearing loss Answer: B- the patient had an atherosclerotic stroke preceded by amaurosis fugax, which is embolization of atheromatous debris from plaque most commonly located at the bifurcation of the carotid artery Answer: B- the patient has cysticercosis. The patient's husband must have eaten a undercooked pork with larva of Taenia solium in the meat (intermediate host). Adults that laid eggs developed in the farmer (definitive host). The eggs were transferred to the wife and they developed into larvae (cysticerci, wife is the intermediate host) in the Gl tract which disseminated to the brain. Answer: A~ the patient has CMV retinitis. It should be treated with ganciclovir. If that does not work, foscamet should be tried. Answer: E- the patient has Lyme's disease with bilateral Bell's palsy. Borrelia burgdorferi is a spirochete transmitted by an Ixodes tick. The skin lesion was erythema chronicum migrans. Answer: B- this is an obstructive hydrocephalus. Answer: C- this is classic amyotrophic lateral sclerosis (ALS) 88

You might also like